Site Loader

Содержание

Таблица удельных сопротивлений проводников — Морской флот

На практике нередко приходится рассчитывать сопротивление различных проводов. Это можно сделать с помощью формул или по данным, приведенным в табл. 1.

Влияние материала проводника учитывается с помощью удельного сопротивления, обозначаемого греческой буквой ? и представляющего собой сопротивление проводника длиной 1 м и площадью поперечного сечения 1 мм2. Наименьшим удельным сопротивлением ? = 0,016 Ом•мм2/м обладает серебро. Приведем среднее значение удельного со п ротивления некоторых проводников:

Серебро – 0,016 , Свинец – 0,21 , Медь – 0,017 , Никелин – 0,42 , Алюминий – 0,026 , Манганин – 0,42 , Вольфрам – 0,055 , Константан – 0,5 , Цинк – 0,06 , Ртуть – 0,96 , Латунь – 0,07 , Нихром – 1,05 , Сталь – 0,1 , Фехраль – 1,2 , Бронза фосфористая – 0,11 , Хромаль – 1,45 .

При различных количествах примесей и при разном соотношении компонентов, входящих в состав реостатных сплавов, удельное сопротивление может несколько измениться.

Сопротивление рассчитывается по формуле:

где R — сопротивление, Ом; удельное сопротивление, (Ом•мм2)/м; l — длина провода, м; s — площадь сечения провода, мм2.

Если известен диаметр провода d, то площадь его сечения равна:

Измерить диаметр провода лучше всего с помощью микрометра, но если его нет, то следует намотать плотно 10 или 20 витков провода на карандаш и измерить линейкой длину намотки. Разделив длину намотки на число витков, найдем диаметр провода.

Для определения длины провода известного диаметра из данного материала, необходимой для получения нужного сопротивления, пользуются формулой

Примечание. 1. Данные для проводов, не указанных в таблице, надо брать как некоторые средние значения. Например, для провода из никелина диаметром 0,18 мм можно приблизительно считать, что площадь сечения равна 0,025 мм2, сопротивление одного метра 18 Ом, а допустимый ток равен 0,075 А.

2. Для другого значения плотности тока данные последнего столбца нужно соответственно изменить; например, при плотности тока, равной 6 А/мм2, их следует увеличить в два раза.

Пример 1. Найти сопротивление 30 м медного провода диаметром 0,1 мм.

Решение. Определяем по табл. 1 сопротивление 1 м медного провода, оно равно 2,2 Ом. Следовательно, сопротивление 30 м провода будет R = 30•2,2 = 66 Ом.

Расчет по формулам дает следующие результаты: площадь сечения провода: s= 0,78•0,12 = 0,0078 мм2. Так как удельное сопротивление меди равно 0,017 (Ом•мм2)/м, то получим R = 0,017•30/0,0078 = 65,50м.

Пример 2. Сколько никелинового провода диаметром 0,5 мм нужно для изготовления реостата, имеющего сопротивление 40 Ом?

Решение. По табл. 1 определяем сопротивление 1 м этого провода: R= 2,12 Ом: Поэтому, чтобы изготовить реостат сопротивлением 40 Ом, нужен провод, длина которого l= 40/2,12=18,9 м.

Проделаем тот же расчет по формулам. Находим площадь сечения провода s= 0,78•0,52 = 0,195 мм2. А длина провода будет l = 0,195•40/0,42 = 18,6 м.

Закон Ома устанавливает связь между силой тока в проводнике и разностью потенциалов (напряжением) на его концах. Формулировка для участка электрической цепи (проводника), не содержащего источников электродвижущей силы (ЭДС): сила тока прямо пропорциональна напряжению и обратно пропорциональна сопротивлению проводника. Законы Ома для замкнутой неразветвлённой цепи: сила тока прямо пропорциональна электродвижущей силе и обратно пропорциональна полному сопротивлению цепи. Закон Ома справедлив для постоянных и квазистационарных токов. Был открыт немецким физиком Георгом Омом в 1826 году. * Современная энциклопедия

В случае переменного тока, величины, входящие в расчётные формулы – становятся комплексными.

Закон Ома в дифференциальной форме – описывает исключительно электропроводящие свойства материала, вне зависимости от геометрических размеров.

Удельное электрическое сопротивление вещества есть электросопротивление изготовленного из него куба со сторонами, равными единице (1метр), когда ток идёт перпендикулярно двум его противоположным граням, площадью 1 квадратный метр каждая. 2/м] (SI – Ω·m, рус. – Ом-метр, англ. – ohm-meter). Для измерения проводниковых материалов разрешается использовать внесистемную единицу –
Ом·мм2/м (для миллиметрового сечения проводника, длиной 1 м., то есть – миллионную часть Ом-метра).

Физический смысл удельного сопротивления: материал (однородный и изотропный*) имеет удельное электрическое сопротивление один Ом·м, если изготовленный из этого материала куб со стороной 1 метр имеет сопротивление 1 Ом при измерении на противоположных гранях куба.
* Изотропность – идентичность физических свойств во всех направлениях.

Удельное сопротивление характеризует способность вещества проводить электрический ток и не зависит от формы и размеров вещества, но меняется, при отличии его температуры от 20 °C (то есть, от комнатной, при которой определялись табличные значения для справочников).

На практике, в технике чаще применяется единица, в миллион раз меньшая (миллиметровое токоведущее сечение), чем Ом·м:

1 мкОм·м (SI – µΩ·m, рус. -9 Ом•м)

Металлы высокой проводимости (не более 0,1 мкОм.м) – используются для изготовления проводов, токопроводящих жил кабелей, обмоток электрических машин и трансформаторов и т. п. Металлы и сплавы высокого сопротивления (не менее 0,3 микроом-метр) – применяются для производства образцовых резисторов, реостатов, электроизмерительных приборов, электронагревательных устройств, нитей ламп накаливания и т. п. Нагревательные сплавы должны выдерживать длительную работу на открытом воздухе – без разрушения при температурах не менее 1000 °С.

Таблица значений удельного электрического сопротивления,
мкОм·м (микроом-метр) = Ом·мм2/м (равные числовые величины)

при температуре окружающей среды 20 градусов по Цельсию

Серебро – 0,015-0,016
Медь – 0,0172-0,0180
Золото – 0,024
Алюминий – 0.026-0.030
Вольфрам – 0,053-0,055
Цинк 0,053-0,062
Никель – 0.068-0,073
Латунь (сплав меди с цинком) – 0,043 – 0,108
Железо – 0,098
Сталь – 0,10-0,14
Олово – 0,12
Оловяно-свинцовый припой – 0,14 – 0,16
Бронзовые сплавы – 0,02 – 0,2
Свинец – 0,217 – 0,227
Никелин – 0,4
Манганин – 0,42 – 0,48
Константан – 0,48 – 0,52
Нихром – 1,05-1,40
Фехраль – 1,15-1,35
Угольно-графитовые щётки для электрических машин – 20-50
Угольный сварочный электрод – 50-90 мкОм·м

Минералка (с минерализацией воды – 2-7 грамм на литр) – 1-4 *10^6 мкОм·м = 1-4 Ом•м
Вода грунтовая – 10-50 *10^6
Влажная / сырая садовая земля (верхний слой почвы, грунта – после поливки) – 20-60 *10^6

Почему в электросетях применяется высокое напряжение

В линии электропередачи, при постоянной передаваемой мощности её потери растут прямо пропорционально длине ЛЭП и обратно пропорционально квадрату ЭДС. Таким образом, считается желательным, увеличение напряжения до величин в десятки (внутригородские воздушные и кабельные сети электропередач на 380 вольт, 6, 10, 20, 35, 110, 220 и 330 кВ) и сотни киловольт (магистральные электросети сверхвысокого – ЛЭП500-750 кВ и ультравысокого напряжения, 1150кВ и выше) на линиях переменного и постоянного (150, 400, 800 кВ) тока. Но, при таких параметрах эксплуатации, постоянно растущем потреблении электрической энергии и частых пиковых перегрузках, износ оборудования, отсутствие резервных мощностей, погодные аномалии, локальные несоответствия требованиям безопасности, непрофессионализм и элементарное разгильдяйство – могут стать причиной нештатных ситуаций и системных аварий (называемых теперь, на английский манер – блэкаут). По этой причине, муниципальные власти любого посёлка и города – имеют постоянную головную боль по обеспечению резервными источниками питания (аккумуляторами и дизель-генераторами) для бесперебойного электроснабжения социальных объектов по резервной схеме.

Спецсплавы на медной основе, в электротехнике

При больших токах, до 10 А – применяют проволочный резистор большой мощности, называемый реостатом. В качестве обмотки используют проволоку, изготовленную из термостабильного (с минимальным температурным коэффициентом) сплава с большим удельным сопротивлением, например, из константана (40% Ni, 1,2% Mn, 58,8% Cu). Если напряжение между соседними витками не превышает 1 вольта – такую проволку можно наматывать плотно, виток к витку, без особой изоляции между витками, благодаря наличию естественной плёнки окисла, образующейся на поверхности данного металла, при быстром (не более трёх секунд) нагреве до достаточно высокой температуры (порядка 900 °С).

В приборах высокого класса точности – применяется манганин (3%Ni, 12%Mn, 85%Cu), менее термоустойчивый, но, в отличие от константанового провода, имеющий очень малую термоЭДС (контактную разность электрических потенциалов) в паре с медью.

Обозначения рекомендуемых кратных и дольных величин от единиц СИ

10^9 Ом – гигаом ГОм GΩ
10^6 Ом – мегаом МОм MΩ
10^3 Ом = 1000 Ом – килоом кОм kΩ. -9 Ом – наноом нОм nΩ

Зависимость сопротивления от температуры.

При нагревании, электрическое сопротивление металлических проводников – возрастает, а при охлаждении – уменьшается. Для вычисления, по формуле, электросопротивления при определённой температуре – используют, так называемый, «температурный коэффициент сопротивления» (ТКС). Расчёты ведутся от некоторого начального уровня температуры. Для интервала температур, в пределах обычных погодных условий (в зимнее и летнее время года) окружающей среды, зависимость для проводника описывается математической формулой:

R2 = R1 * (1 + α * (t2 – t1)),

где R1 (начальное, известное значение, при нуле или 20 градусов по Цельсию, измеренное или посчитанное) и R2 (искомое) – сопротивления резистора соответственно при температурах t1 (0°С или 20°С) и t2; α – температурный коэффициент сопротивления (из справочной таблицы), равный относительному изменению электр. сопротивления (удельного или абсолютного) при изменении температуры на 1 °С. Так как значения ТКС очень малы, то в справочниках их указывают в единицах тысячных или миллионных долей (ppm/°С – Parts Per Million) относительного изменения сопротивления на градус.

Обычно, исходные, табличные значения различных физических постоянных – приводятся или к нормальной комнатной температуре +20 °С или к нулевой (в справочных таблицах проводниковых и реостатных материалов, применяемых в электрических аппаратах).

В металлических термометрах, изготавливаемых из медной или платиновой проволоки – электросопротивление, с повышением температуры (без экстремально высоких, для этих материалов, значений) увеличивается почти линейно. Но, при чрезмерно сильном нагреве, к примеру, тонкого медного провода до температуры красного каления, его активное электрическое сопротивление постоянному току возрастает многократно.

Пример расчёта для стометрового алюминиевого шинопровода, радиусом 40 мм, нагретого на 95°С:
R = (R1 * (1 + α * (t2–t1))) * L / S =
= 2,62*10 -8 Ом•м * (1 + 0,0042*95) * 100 / (3,14 * 40 2 * 10 -6 ) = 7,3 * 10 -4 Ом
где:
S – площадь сечения в м 2 (с вычетом толщины слоёв изоляции),
L – длина проводника в метрах.

Температурный коэффициент сопротивления х10 -3 , 1/градус:
Алюминий – 4,2
Бронза оловянистая твёрдотянутая – 0,6-0,7
Вольфрам – 4,2
Графит – -1,3
Дюраль – 2,2
Константан – 0,003-0,005
Латунь – 1,5
Манганин – 0,03-0,06 (при температуре до 250-300°С)
Медь – 4,3
Нихром – 0,14
Серебро – 4,0
Сталь – 9,0
Цинк – 4,2

2

Постоянные резисторы и их маркировка

В буквенно-цифровой (кодовой) маркировке резисторов – на их корпус наносится числовое значение электрического сопротивления и буквы, первая из которых обозначает множитель (R или Е – Ом,&nbsp K – килоом,&nbsp M – мегаом) и, заодно, определяет положение разделительной запятой десятичного знака. 2)/4
с помощью своего калькулятора, находится диаметр (в миллиметрах) = корень квадратный из (4 * S / 3.14)

Длина провода, в единицах системы СИ (переводим в метры):
80 см = 0.8 м

Находим электр. сопротивление по формуле:
R = (p * L) / S = (0.017 * 0.8) / 0.2 = 0.068 Ом

Ответ: с точностью до второго знака после запятой, R = 0.07 Ом

Электромонтажные работы – монтаж электрики, подключение и обслуживание электропроводки. | Минисправочник по электрическим параметрам: соотношения Ом х мм2/м и мкОм x м (микроом), в технических расчётах.

Дата публикации: 26 марта 2013 .
Категория: Электротехника.

При замыкании электрической цепи, на зажимах которой имеется разность потенциалов, возникает электрический ток. Свободные электроны под влиянием электрических сил поля перемещаются вдоль проводника. В своем движении электроны наталкиваются на атомы проводника и отдают им запас своей кинетической энергии. Скорость движения электронов непрерывно изменяется: при столкновении электронов с атомами, молекулами и другими электронами она уменьшается, потом под действием электрического поля увеличивается и снова уменьшается при новом столкновении. В результате этого в проводнике устанавливается равномерное движение потока электронов со скоростью нескольких долей сантиметра в секунду. Следовательно, электроны, проходя по проводнику, всегда встречают с его стороны сопротивление своему движению. При прохождении электрического тока через проводник последний нагревается.

Электрическое сопротивление

Электрическим сопротивлением проводника, которое обозначается латинской буквой r, называется свойство тела или среды превращать электрическую энергию в тепловую при прохождении по нему электрического тока.

На схемах электрическое сопротивление обозначается так, как показано на рисунке 1, а.

0,050,070,10,20,30,40,50,711,522,54611
Наибольший допустимый ток, А0,711,32,53,545710141720253054
Рисунок 1. Условное обозначение электрического сопротивления

Переменное электрическое сопротивление, служащее для изменения тока в цепи, называется реостатом. На схемах реостаты обозначаются как показано на рисунке 1, б. В общем виде реостат изготовляется из проволоки того или иного сопротивления, намотанной на изолирующем основании. Ползунок или рычаг реостата ставится в определенное положение, в результате чего в цепь вводится нужное сопротивление.

Длинный проводник малого поперечного сечения создает току большое сопротивление. Короткие проводники большого поперечного сечения оказывают току малое сопротивление.

Если взять два проводника из разного материала, но одинаковой длины и сечения, то проводники будут проводить ток по-разному. Это показывает, что сопротивление проводника зависит от материала самого проводника.

Температура проводника также оказывает влияние на его сопротивление. С повышением температуры сопротивление металлов увеличивается, а сопротивление жидкостей и угля уменьшается. Только некоторые специальные металлические сплавы (манганин, констаитан, никелин и другие) с увеличением температуры своего сопротивления почти не меняют.

Итак, мы видим, что электрическое сопротивление проводника зависит от: 1) длины проводника, 2) поперечного сечения проводника, 3) материала проводника, 4) температуры проводника.

За единицу сопротивления принят один Ом. Ом часто обозначается греческой прописной буквой Ω (омега). Поэтому вместо того чтобы писать «Сопротивление проводника равно 15 Ом», можно написать просто: r = 15 Ω.
1 000 Ом называется 1 килоом (1кОм, или 1кΩ),
1 000 000 Ом называется 1 мегаом (1мгОм, или 1МΩ).

При сравнении сопротивления проводников из различных материалов необходимо брать для каждого образца определенную длину и сечение. Тогда мы сможем судить о том, какой материал лучше или хуже проводит электрический ток.

Видео 1. Сопротивление проводников

Удельное электрическое сопротивление

Сопротивление в омах проводника длиной 1 м, сечением 1 мм² называется удельным сопротивлением и обозначается греческой буквой ρ (ро).

В таблице 1 даны удельные сопротивления некоторых проводников.

Удельные сопротивления различных проводников

Материал проводникаУдельное сопротивление ρ в
Серебро
Медь
Алюминий
Вольфрам
Железо
Свинец
Никелин (сплав меди, никеля и цинка)
Манганин (сплав меди, никеля и марганца)
Константан (сплав меди, никеля и алюминия)
Ртуть
Нихром (сплав никеля, хрома, железа и марганца)
0,016
0,0175
0,03
0,05
0,13
0,2
0,42
0,43
0,5
0,94
1,1

Из таблицы видно, что железная проволока длиной 1 м и сечением 1 мм² обладает сопротивлением 0,13 Ом. Чтобы получить 1 Ом сопротивления нужно взять 7,7 м такой проволоки. Наименьшим удельным сопротивлением обладает серебро. 1 Ом сопротивления можно получить, если взять 62,5 м серебряной проволоки сечением 1 мм². Серебро – лучший проводник, но стоимость серебра исключает возможность его массового применения. После серебра в таблице идет медь: 1 м медной проволоки сечением 1 мм² обладает сопротивлением 0,0175 Ом. Чтобы получить сопротивление в 1 Ом, нужно взять 57 м такой проволоки.

Химически чистая, полученная путем рафинирования, медь нашла себе повсеместное применение в электротехнике для изготовления проводов, кабелей, обмоток электрических машин и аппаратов. Широко применяют также в качестве проводников алюминий и железо.

Сопротивление проводника можно определить по формуле:

где r – сопротивление проводника в омах; ρ – удельное сопротивление проводника; l – длина проводника в м; S – сечение проводника в мм².

Пример 1. Определить сопротивление 200 м железной проволоки сечением 5 мм².

Пример 2. Вычислить сопротивление 2 км алюминиевой проволоки сечением 2,5 мм².

Из формулы сопротивления легко можно определить длину, удельное сопротивление и сечение проводника.

Пример 3. Для радиоприемника необходимо намотать сопротивление в 30 Ом из никелиновой проволоки сечением 0,21 мм². Определить необходимую длину проволоки.

Пример 4. Определить сечение 20 м нихромовой проволоки, если сопротивление ее равно 25 Ом.

Пример 5. Проволока сечением 0,5 мм² и длиной 40 м имеет сопротивление 16 Ом. Определить материал проволоки.

Материал проводника характеризует его удельное сопротивление.

По таблице удельных сопротивлений находим, что таким сопротивлением обладает свинец.

Выше было указано, что сопротивление проводников зависит от температуры. Проделаем следующий опыт. Намотаем в виде спирали несколько метров тонкой металлической проволоки и включим эту спираль в цепь аккумулятора. Для измерения тока в цепь включаем амперметр. При нагревании спирали в пламени горелки можно заметить, что показания амперметра будут уменьшаться. Это показывает, что с нагревом сопротивление металлической проволоки увеличивается.

У некоторых металлов при нагревании на 100° сопротивление увеличивается на 40 – 50 %. Имеются сплавы, которые незначительно меняют свое сопротивление с нагревом. Некоторые специальные сплавы практически не меняют сопротивления при изменении температуры. Сопротивление металлических проводников при повышении температуры увеличивается, сопротивление электролитов (жидких проводников), угля и некоторых твердых веществ, наоборот, уменьшается.

Способность металлов менять свое сопротивление с изменением температуры используется для устройства термометров сопротивления. Такой термометр представляет собой платиновую проволоку, намотанную на слюдяной каркас. Помещая термометр, например, в печь и измеряя сопротивление платиновой проволоки до и после нагрева, можно определить температуру в печи.

Изменение сопротивления проводника при его нагревании, приходящееся на 1 Ом первоначального сопротивления и на 1° температуры, называется температурным коэффициентом сопротивления и обозначается буквой α.

Если при температуре t сопротивление проводника равно r, а при температуре t равно rt, то температурный коэффициент сопротивления

Примечание. Расчет по этой формуле можно производить лишь в определенном интервале температур (примерно до 200°C).

Приводим значения температурного коэффициента сопротивления α для некоторых металлов (таблица 2).

Значения температурного коэффициента для некоторых металлов

Серебро
Медь
Железо
Вольфрам
Платина0,0035
0,0040
0,0066
0,0045
0,0032Ртуть
Никелин
Константан
Нихром
Манганин0,0090
0,0003
0,000005
0,00016
0,00005

Из формулы температурного коэффициента сопротивления определим rt:

Пример 6. Определить сопротивление железной проволоки, нагретой до 200°C, если сопротивление ее при 0°C было 100 Ом.

Пример 7. Термометр сопротивления, изготовленный из платиновой проволоки, в помещении с температурой 15°C имел сопротивление 20 Ом. Термометр поместили в печь и через некоторое время было измерено его сопротивление. Оно оказалось равным 29,6 Ом. Определить температуру в печи.

Электрическая проводимость

До сих пор мы рассматривали сопротивление проводника как препятствие, которое оказывает проводник электрическому току. Но все же ток по проводнику проходит. Следовательно, кроме сопротивления (препятствия), проводник обладает также способностью проводить электрический ток, то есть проводимостью.

Чем большим сопротивлением обладает проводник, тем меньшую он имеет проводимость, тем хуже он проводит электрический ток, и, наоборот, чем меньше сопротивление проводника, тем большей проводимостью он обладает, тем легче току пройти по проводнику. Поэтому сопротивление и проводимость проводника есть величины обратные.

Из математики известно, что число, обратное 5, есть 1/5 и, наоборот, число, обратное 1/7, есть 7. Следовательно, если сопротивление проводника обозначается буквой r, то проводимость определяется как 1/r. Обычно проводимость обозначается буквой g.

Электрическая проводимость измеряется в (1/Ом) или в сименсах.

Пример 8. Сопротивление проводника равно 20 Ом. Определить его проводимость.

Если r = 20 Ом, то

Пример 9. Проводимость проводника равна 0,1 (1/Ом). Определить его сопротивление,

Если g = 0,1 (1/Ом), то r = 1 / 0,1 = 10 (Ом)

Источник: Кузнецов М. И., «Основы электротехники» – 9-е издание, исправленное – Москва: Высшая школа, 1964 – 560с.

Сопротивление удельное металлов — Справочник химика 21


    Сплавы облада от большим остаточным сопротивлением, причем для многих (нержавеющая сталь, монель, мельхиор и др.) р яа Ро и слабо зависит от температуры (табл. 3.14). Упругая и пластическая дефор-ма 1ия заметно изменяют удельное электрическое сопротивление чистых металлов (рис. 3.16) и практически не влияют на сопротивление сплавов. Это свойство чистых метал- [c.235]

    Удельное электрическое сопротивление чистых металлов при значительном наклепе возрастает приблизительно на 2-6 %. Увеличение электрического сопротивления металлов можно объяснить прежде всего тем, что при наклепе искажается пространственная решетка кристаллов. На значение электрического сопротивления влияет также и изменение межатомных связей, вызванных наклепом. Эго изменение приводит также к увеличению межатомных расстояний. [c.57]

    ВАВИСИМОСТЬ УДЕЛЬНОГО СОПРОТИВЛЕНИЯ ЧИСТЫХ МЕТАЛЛОВ ОТ ТЕМПЕРАТУРЫ 

[c.932]

    Печи сопротивления. Нагрев металла сопротивлением осуществляется прохождением электрического тока через металл. Печи сопротивления обычно применяются для тугоплавких металлов. Электрооборудование этих печей дешевле, чем индукционных. Греющий элемент должен иметь возможно большее удельное сопротивление. Греющими элементами могут служить уголь, графит, криптол (зернистый уголь), карборунд, тугоплавкие металлы. В таких печах можно [c.341]

    Для практического осуществления электронагревания пользуются или проводниками из материалов с большим удельным сопротивлением (некоторые металлы, уголь и т. п.) или электрической дугой, которая по существу есть тоже проводник с очень большим сопротивлением. Электрическая дуга позволяет получать большое падение потенциала, а следователыю, и весьма высокую температуру на очень коротком участке цепи. 

[c.25]

    Магнитная восприимчивость и удельное сопротивление некоторых металлов [c.453]

    Электрические свойства карбида кремния определяются тем, что он относится к группе электронных полупроводников. Этим обстоятельством, в частности, объясняется то, что электрическое сопротивление карбида кремния характеризуется цифрами, промежуточными между значениями сопротивления типичных металлов (проводников) и типичных изоляторов. Удельное электрическое сопротивление 51С лежит в пределах от 1 до 10 ом см, см ) [13, 14, 15]. Приведенные цифры характеризуют линейную (или приближенно линейную) проводимость 

[c.130]

    ЗАВИСИМОСТЬ УДЕЛЬНОГО СОПРОТИВЛЕНИЯ ЧИСТЫХ МЕТАЛЛОВ ОТ ТЕМПЕРАТУРЬ [c.933]

    Значения удельного сопротивления и температурного коэффициента сопротивления для ряда материалов приведены в табл. 1, а на рнс. 12 показан характер изменения удельного сопротивления некоторых металлов в зависимости от температуры. [c.35]

    П1-2- Удельное сопротивление р металлов и сплавов, применяемых в нагревательных устройствах [c.41]

    Удельное электрич. сопротивление токопроводящего металла кабеля Рк Р 1 Ом мм /м ( 

[c.34]

    Величину К можно рассчитать теоретически, но для этого надо знать эмиссионную способность и удельное сопротивление данного металла. Чаще величину К определяют на экспериментальном аппарате, в котором можно найти зависимость между У и / и изготовить регулирующие приборы, которые будут поддерживать постоянство величины и, таким образом, обеспечат устойчивую температуру. Так были созданы специальные автоматические регулирующие устройства — так называемые БАУ (блоки автоматического управления), широко используемые Б технологии титана и циркония. [c.322]

    Величину К можно рассчитать теоретически, но для этого надо знать эмиссионную способность и удельное сопротивление данного металла Чаще величину К определяют на экспериментальном аппарате, в котором можно найти зависимость между 

[c.322]

    Зависимость удельного электрического сопротивления чистых металлов (а) слюды (б) от температуры i — свинец г — железо 3 — медь. [c.765]

    Удельное сопротивление металлических слоев, полученных методом катодного распыления, значительно больше, чем удельное сопротивление массивного металла, однако, благодаря чрезвычайно хорошему охлаждению металла в тонких слоях, последние могут выдерживать чрезмерные плотности тока при -затяжке. [c.75]


    Удельное сопротивление чистых металлов в области высоких температур примерно пропорционально температуре, т. е. изменение удельного сопротивления на один градус почти постоянно. В области низких температур скорость изменения удельного сопротивления снижается, а при самых низких температурах удельное сопротивление металла приближается к постоянной величине. Поэтому термометр сопротивления из металла является плохим термометром при температурах жидкого гелия. В термометрах сопротивления иногда используют сплавы [55, 87, 121.  
[c.299]

    Удельное сопротивление некоторых металлов при комнатной [c.136]

    Поверхность металлов обычно покрыта окислами, удельное сопротивление которых намного больше сопротивления самого металла это сопротивление, однако, тем меньше, чем выше температура. В точках соприкосновения при прохождении тока развивается тепло, здесь-то и переходит ток с одной поверхности на другую. В контактах металл — металл число точек соприкосновения обычно больше, чем в контактах металл—графит при повышении плотности тока в отдельных точках этого контакта быстро повышается температура, углерод выгорает и контакт портится. С увеличением давления на поверхность контакта сопротивление его падает. 

[c.71]

    Электролиты проводят ток значительно хуже, чем металлы, т. е. обладают при тех же размерах большим сопротивлением. Удельное сопротивление электролита р определяют по таблицам или по кривым, приведенным в книгах по гальваностегии и соответствующих справочниках. Ниже для примера приведено удельное 

[c.12]

    Электромагнитные насосы, являющиеся электрическими машинами, имеют все же более низкий к. п. д., чем электрические вращающиеся машины соответствующей мощности. Это объясняется, прежде всего, более высоким удельным сопротивлением жидких металлов по сравнению, например, с медью, неизбежной [c.28]

    При выборе соотношения сечений термоэлектродов для данной термопары следует учитывать, что коэффициент теплопроводности и удельные сопротивления разных металлов и сплавов существенно различны, поэтому оптимальное сечение термоэлектродов в одних и тех же условиях также должно быть различным. Сечения термоэлектродов термопары рекомендуется выбирать такими, чтобы они были пропорциональны квадратным корням их удельных сопротивлений и обратно пропорциональны квадратным корням их коэффициентов теплопроводности. Из этих соображений в случае, например, термопары медь — константан медную проволоку лучше брать значительно меньшего сечения, чем константа-новую. 

[c.154]

    Путем катодного распыления удается получать пленки тугоплавких металлов. Для получения нитридов тугоплавких металлов применяется разряд в смеси аргона с азотом, для получения карбидов — смесь аргона с метаном или аргона с окисью углерода. Поскольку такие металлы, как титан, тантал, цирконий и ниобий, являются хорошими газопоглотителями, то даже при распылении в атмосфере аргона без специальной добавки ре-а 1(тивного газа образуются пленки, удельное электрическое сопротивление которых больше, чем удельное сопротивление распыляемого металла. Эти пленки имеют такую же структуру, как и сам распыляемый металл, а растворенные в них атомы газов, не вытесняя атомов металла из кристаллической решетки, располагаются в промежутках между ее узлами. 

[c.21]

    Под толщиной металлической пленки в данном случае понимают ту толщину, которую имел бы слой, если бы его сопротивление было равно удельному сопротивлению массивного металла. На самом деле лишь для некоторых пленок их проводимость мало отличается от проводимости исходного материала. 

[c.255]

    Устройство для индукционного нагрева металлов в самом общем виде представляет собой обмотку, питаемую переменным током, В переменном магнитном поле, создаваемом этой обмоткой, называемой индуктором, помещается нагреваемое металлическое тело. Переменный магнитный поток возбуждает в металлическом теле переменную э. д. с. ц вихревые токи, которые и нагревают тело. Таким образом, теплота, выделяющаяся в теле, зависит, помимо других факторов, от удельного сопротивления нагреваемого металла. В частности, в непроводниках ток проводимости не возникает и джоулево тепло не выделяется, что позволяет при индукционном нагреве выделять энергию исключительно в нагреваемом металле. 

[c.8]

    Как видно из выражений (1-16д), (1-16е) и др., выделение энергии в металле тем меньше, чем меньше его удельное сопротивление. Поэтому металлы с малым удельным сопротивлением иногда (ч. И) выплавляют в тиглях из графита или металлов с достаточно большим удельным сопротивлением, например из стали. [c.32]

    При образовании твердого раствора электропроводность металла снижается. При размещении в пространственной решетке растворителя чуждых атомов растворенного вещества электрическое поле решетки растворителя искажается, и рассеяние элеюронов увеличивается. Электрические свойства твердого раствора обусловлены также химическим взаимодействием компонентов. При наклепе удельное электрическое сопротивление твердых растворов, так же как и чистых металлов, повьш1ается, а при отжиге понижается. При наклепе и отжиге твердых растворов, даже слабо-концентрирюванных, их электрическое сопротивление изменяется в большей степени, чем сопротивление чистых металлов в тех же условиях. 

[c.58]

    Физические свойства. Металлический ванадий — блестящий металл серо-стального цвета. Ванадий — один из наиболее твердых металлов, тверже стали и кварца. Он хорошо шлифуется и полируется, причем его отполированная поверхность долгое время сохраняет блеск. Наиболее чистые образцы V достаточно ковки, тягучи, но после нагревания с водородом становятся хрупкими. Металлический ванадий поддается намагничиванию, Удельное сопротивление холодного металла 26 10 ом1смР см удельная теплоемкость 0,120 кал град- г (в интервале 20—100° С). Ванадий в расплавленном состоянии не обладает заметной летучестью даже в высоком вакууме. [c.305]

    В табл. 19.1 представлены значения удельного сопротивления и температурного коэффициента сопротивления чистых металлов, а также, в некоторых случаях, отношение удельного сопротивления при температуре жидкого гелия к удельному сопротивлению при нормальных условиях, р4,2 >к/р273°к, характеризующее достигнутую степень чистоты материала. В тех случаях, когда для данного металла приводятся более подробные данные, соответствующее указание дается в первом столбце таблицы. Металлы в таблице расположены в порядке возрастания массового числа. [c.304]

    При частотах 10 гц (и выше) удельное сопротивление некоторых металлов, используемых в качестве вводов (ковар),становится недопустимо высоким. Для снижения удельного сопротивления коваровые вводы, работающие в области высоких частот, обязательно покрывают медью или золотом. [c.274]

    Интересны цифры, характеризующие удельное электросопротивление гексаборидов они, как правило, меньше, чем сопротивление чистых металлов (см. табл. 29). Г. В Самсонов и Ю. Б. Падерно [743] объясняют это тем, что электроны бсра восполняют пробелы в недостроенной оболочке лантанида и тем самым снижают его электросопротивление. [c.283]

    Окклюзия газов металлами является важным разделом в новом учении О материалах. Окклюдированные газы могут существенно влиять на механические, физические и коррозионные свойства металлов. В течение последних пятнадцати лет стало очевидным, что пластичными можно получить сплавы, например сплавы Т1, Nb, Сг, Мо и , только при малом остаточном содержании газа. При большом содержании газов у этих и других металлов IV, V и VI групп изменяются такие физические свойства как магнитная восприимчивость, электрическое сопротивление, удельная теплоемкость и сверхпроводимость. Для сплавов 2г сопротивляемость коррозии в воде при повышенных температурах изменяется при ок-клюдировании даже небольшого количества водорода, образующегося в результате окисления металла водой. Наличие окклюдированных газов в металлах по-разному влияет на их рабочие характеристики. Поэтому для правильного использования металлов в промышленности необходимо не только знать, каким образом в разных условиях изменяются свойства металлов, содержащих окклюдированные газы, по и ясно понимать процесс окклюзии. [c.202]

    Эти обстоятельства (необходимость достаточно высокого удельного электрического сопротивления расплавляемого металла и наличие в большинстве случаев конденсаторной батареи и преобразователей частоты) ограничивают области применения индукционных печей без сердечника в этих печах рационально плавить или специальные сорта стали, которые невозможно или неэкономично (из-за сложности технологического процесса) плавить в дуговых печах, или такие металлы или сплавьи, высокая стоимость которых позволяет пренебречь пониженным электрическим к. п. д. этих печей (если экономия на угаре ценного металла в индукционных печах компенсирует понижение [c.177]


Сплавы для проводников и элементов сопротивления :: Технология металлов

Проводниковые материалы должны изготовляться из возможно чистых металлов, а материалы высокого сопротивления — из сплавов, структура которых пред­ставляет твердый раствор.

Основными проводниковыми материалами являются медь и алюминий. Понятие «чистого» металла условно, так как чистота получаемых металлов повы­шается из года в год по мере роста технических средств в производстве и в лабораториях. С другой стороны, электрическое сопротивление чистых металлов резко возрастает при наличии малых количеств растворенных примесей. Из рис.  видно, как значительно падает проводимость меди при введении в ее состав таких примесей, как кремний, железо, мышьяк, бериллий в количествах <0,2%. Для проводниковой техники разработан интернациональный стан­дарт технически чистой отожженной меди (1913 г.), удельное сопротивление которой при 20° С равно 1,7241 мкОм-см. С тех пор добились дальнейшего очи­щения меди и повышения ее проводимости.

Для повышения прочности медного провода в его состав вводят примеси Cd, Sn, Αl, Ρ, Cr, Be. Электропроводность при этом, естественно, понижается.

Рис. 1. Влияние примесей на электропроводность меди

Наиболее распространенной является так называемая кадмиевая бронза (0,9% Cd, остальное медь), которая в твердотянутом состоянии обладает проводимостью до 90% от проводимости меди при временном сопротивлении при растяжении, в 2—2,5 раза большем, чем у меди. В табл. 25 приведены составы и свойства медных проводниковых материалов.

Проводниковый алюминий имеет электропроводность, равную 65% от про­водимости меди. Однако ввиду малой плотности алюминия его проводимость, рассчитанная на 1 кг проводника, составляет 214% от таковой для меди. Это позволяет облегчить конструкцию опор на линиях передач с алюминиевыми про­водами. Для повышения прочности алюминиевых проводов в их состав вводят магний и кремний (совместно), которые образуют соединение Mg2Si, очень мало растворимое в алюминии при комнатной температуре (меньше 0,25%). Путем закалки и старения удается повысить временное сопротивление такого сплава в 2 раза при электропроводности 90% от проводимости чистого алюминия. Сплавы такого типа (альдрей и альмелек) содержат 0,4 и 0,7% Mg, 0,5—0,6% Si и до 0,3% Fe. Альдрей (0,4% Mg, 0,6% Si и 0,3% Fe) имеет температурный коэф­фициент электрического спротивления, равный 3,6 * 10-4, т. е. весьма близкий к температурному коэффициенту чистого алюминия (4,0 * 10-4). Это еще раз свидетельствует о том, что при обработке путем дисперсионного твердения Mg2Si выделяется из раствора почти нацело.

Для реостатов в нагревательных приборах, а также там, где необходимо высокое электрическое сопротивление и малый температурный коэффициент, применяют сплавы железа с примесями, образующими твердые растворы. В табл. 2 приводятся типичные железные сплавы, а также для сравнения сплавы на никелевой основе.

Эти сплавы являются не только сплавами высокого сопротивления, но и жаростойкими. Для придания жаростойкости в железные сплавы вводятся хром и алюминий. Железные сплавы дешевле никелевых, однако они не только не яв­ляются заменителями, но имеют также и более высокую рабочую температуру.

В качестве элементов сопротивления применяются  медноникелевые сплавы— константан и никелин. Рабочая температура константана до 400° С, никелина — до 200° С.

Таблица 1

Медные  проводниковые  сплавы

Сплав

Состояние

Электро­провод­ность, %

Временное сопротивле­ние при рас­тяжении, МПа

Удлине­ние,  %

Чистая медь

Отожженная

101

220—270

50

Твердотянутая

98

До 480

4

Кадмиевая     бронза

(0,9% Cd)

Отожженная

95

310—380

50  

Твердотянутая

83—90

До 730

4

Бронза     (0,75% Sn

или 0,8% Cd и 0,6% Sn)

Отожженная

55—60

290 .

55

Твердотянутая

50—55

До 730

4

Бронза      (2,5% А1, 2% Sn)

Отожженная

15—18

370

45

Твердотянутая

15—18

До 970

4

Фосфористая  бронза

(7% Sn, 0,1% Ρ)

Отожженная

10—15

400

60

Твердотянутая

10—15

1050

3

Таблица 2

Сплавы  для   реостатов  и  нагревательных  приборов

Сплав

Состав (средний),  %

Удельное электро­сопроти­вление, мкОм · см

Темпера­турный коэффи­циент α

Наивыс­шая ра­бочая темпера­тура, °С

Х13Ю4 (фехраль)

13,5 Сr; 4,5 Аl; остальное Fe

126

0,00005

1000

0Х23Ю5

23 Сr; 5 Аl; остальное Fe

137

1200

0Х27Ю5А

27 Сr; 5,5 Аl; остальное Fe

142

0,00002

1300

Сверхмегапир

37 Сr; 7,5 Аl; остальное Fe

180

0,00012

1350

Х15Н60 (нихром)

16,5 Сr: 58 Ni: остальное Fe

110

0,00017

1000

Х20Н80

21,5 Сr; остальное Ni

100

1100

 

Таблица 3.

Влияние   различных  элементов   на  удельное электросопротивление  железа

Элемент

 

 

Пределы концентрации и

температуры

Средние  значения   возрастания

удельного сопротивления, мкОм.см

% (по массе)

°с

на 1 % (по массе)

на 1 % (ат.)

Аl

0—2,0

18—23

11,1—14,4

6,0—7,7

As

0—2,6

6,8

9,10

Au

1.1

5,80

В

0—0,45

6,2

1,25

С

0—0,9

20

34,0

7,6

Со

0,5

18—30

1,0—3,0

1,1—3,2

Сг

0,3

12

2,5—5,4

2,3—5,0

Сu

0—1

3,0—4,0

3,4—4,6

Μn

0—2

18—30

5,0—10,5

4,9—10,3

Mo

0—1

17

3,4

5,8

Ν

0—0,1

20

14,6

3,8

Ni

0—5

18—30

1,55—4,45

1,7-4,7

Ρ

0—0,3

11,4

6,1

S

0—0,1

20

12,0

6,9

Si

0—1

20

13—15,8

6,5—8,0

Ti

1,0

0,9

V

0—1

6,7

6,1

W

0—2

15—20

2,0—3,6

6,5—11,8

 

 

 

Источник:
Лившиц Б.Г., Крапошин В.С, Липецкий Я.Л. «Физические свойства металлов и сплавов». М. «Металлургия», 1980.

Удельное сопротивление. Реостаты — урок. Физика, 8 класс.

Для рассмотрения характеристик электрических параметров рассмотрим назначение приборов:

  1. сила тока в цепи определяется амперметров, который подключается последовательно с соблюдением полярности;
  2. напряжение на участке цепи измеряется вольтметром, который подключается параллельно к тому участку или прибору, на котором нужно узнать разность потенциалов или напряжения;
  3. на деревянной изолирующей подставке — устройство, имеющее провода с различными значениями сопротивления;
  4. значение тока можно регулировать реостатом.

 

Эксперимент 1. В цепь источника тока по очереди будем включать различные проводники, например, никелиновые проволоки одинаковой толщины, но разной длины. Выполнив указанные опыты, мы установим, что из двух никелиновых проволок одинаковой толщины более длинная проволока имеет большее сопротивление.
Эксперимент 2. По очереди будем включать никелиновые проволоки одинаковой длины, но разной толщины (разной площади поперечного сечения). Сопротивление проводника обратно пропорционально площади поперечного сечения проводника: чем тоньше проводник, тем больше сопротивление.

Эксперимент 3. Изучим сопротивление двух проводников равной длины и толщины из разных материалов (никель и нихром). Сопротивление проводников отличается.

Впервые зависимость сопротивления проводника от вещества, из которого он изготовлен, и от длины проводника обнаружил немецкий физик Георг Ом. Он установил:

Сопротивление проводника напрямую зависит от его длины и материала,  но обратным образом зависит от площади поперечного сечения проводника.

 

Обрати внимание!

Из этого можно сделать вывод: чем длиннее проводник, тем больше его электрическое сопротивление.
Сопротивление проводника обратно пропорционально площади его поперечного сечения, т.е. чем толще проводник, тем его сопротивление меньше, и, наоборот, чем тоньше проводник, тем его сопротивление больше.

Чтобы лучше понять эту зависимость, представьте себе две пары сообщающихся сосудов, причём у одной пары сосудов соединяющая трубка тонкая, а у другой — толстая. Ясно, что при заполнении водой одного из сосудов (каждой пары) переход её в другой сосуд по толстой трубке произойдёт гораздо быстрее, чем по тонкой, т.е. толстая трубка окажет меньшее сопротивление течению воды. Точно так же и электрическому току легче пройти по толстому проводнику, чем по тонкому, т.е. первый оказывает ему меньшее сопротивление, чем второй.

 

Удельное сопротивление проводника зависит от строения вещества. Электроны при движении внутри металлов взаимодействуют с атомами (ионами), находящимися в узлах кристаллической решётки. Чем выше температура вещества, тем сильнее колеблются атомы и тем больше удельное сопротивление проводников.

Удельное электрическое сопротивление — физическая величина \(\rho\), характеризующая свойство материала оказывать сопротивление прохождению электрического тока:
ρ=R⋅Sl, где удельное сопротивление проводника обозначается греческой буквой \(\rho\) (ро), \(l\) — длина проводника, \(S\) — площадь его поперечного сечения.

Определим единицу удельного сопротивления. Воспользуемся формулой ρ=R⋅Sl.

Как известно, единицей электрического сопротивления является \(1\) Ом, единицей площади поперечного сечения проводника — \(1\) м², а единицей длины проводника — \(1\) м. Подставляя в формулу, получаем:

1 Ом ⋅1м21 м=1 Ом ⋅1 м, т.е. единицей удельного сопротивления будет Ом⋅м.

 

На практике (например, в магазине при продаже проводов) площадь поперечного сечения проводника измеряют в квадратных миллиметрах, В этом случае единицей удельного сопротивления будет:

1 Ом ⋅1мм21 м, т.е. Ом⋅мм2м.

В таблице приведены значения удельного сопротивления некоторых веществ при \(20\) °С.

 

Удельное сопротивление увеличивается пропорционально температуре.

При нагревании колебания ионов металлов в узлах металлической решётки увеличиваются, поэтому свободного пространства для передвижения электронов становится меньше. Электроны чаще отбрасываются назад, поэтому значение тока уменьшается, а значение сопротивления увеличивается.

 

Обрати внимание!

Из всех металлов наименьшим удельным сопротивлением обладают серебро и медь. А это значит, что медь и серебро лучше остальных проводят электрический ток.

При проводке электрических цепей, например, в квартирах не используют серебро, т.к. это дорого. Зато используют медь и алюминий, так как эти вещества обладают малым удельным сопротивлением.
Порой необходимы приборы, сопротивление которых должно быть большим. В этом случаем необходимо использовать вещество или сплав с большим удельным сопротивлением. Например, нихром.

Полиэтилен, дерево, стекло и многие другие материалы отличаются очень большим удельным сопротивлением. Поэтому они не проводят электрический ток. Такие материалы называют диэлектриками или изоляторами.

 

Очень часто нам приходится изменять силу тока в цепи. Иногда мы ее увеличиваем, иногда уменьшаем. Водитель трамвая или троллейбуса изменяет силу тока в электродвигателе, тем самым увеличивая или уменьшая скорость транспорта. 

Реостат — это резистор, значение сопротивления которого можно менять.

Реостаты используют в цепи для изменения значений силы тока и напряжения.

Реостат на рисунке состоит из провода с большим удельным сопротивлением (никелин, нихром), по которому передвигается подвижный контакт \(C\) по длине провода, плавно изменяя сопротивление реостата. Сопротивление такого реостата пропорционально длине провода между подвижным контактом \(C\) и неподвижным \(A\). Чем длиннее провод, тем больше сопротивление участка цепи и меньше сила тока. С помощью вольтметра и амперметра можно проследить эту зависимость.


 

На школьных лабораторных занятиях используют переменное сопротивление — ползунковый реостат.

 

 

Он состоит из изолирующего керамического цилиндра, на который намотан провод с большим удельным сопротивлением. Витки проволоки должны быть изолированы друг от друга, поэтому либо проволоку обрабатывают графитом, либо оставляют на проволоке слой окалины. Сверху над проволочной обмоткой закреплен металлический стержень, по которому  перемещается ползунок. Контакты ползунка плотно прижаты в виткам и при движении изолирующий слой графиты или окалины стирается, и тогда электрический ток может проходить от витков проволоки к ползунку, через него подводиться к стержню, имеющему на конце зажим \(1\).

Для соединения реостата в цепь используют зажим \(1\) и зажим \(2\). Ток, поступая через зажим \(2\), идёт по никелиновой проволоке и через ползунок подаётся на зажим \(1\). Перемещая ползунок от \(2\) к \(1\), можно увеличивать длину провода, в котором течёт ток, а значит, и сопротивление реостата.


В электрических схемах реостат изображается следующим образом:


 

Как и любой электрический прибор, реостат имеет допустимое значение силы тока, свыше которого прибор может перегореть. Маркировка реостата содержит диапазон его сопротивления и максимальное допустимое значение силы тока.

Обрати внимание!

Сопротивление реостата нужно учитывать в параметрах электрической цепи. При минимальных значениях сопротивления ток в цепи может вывести из строя амперметр.

Существуют реостаты, в которых переключатель подключается на проводники заданной длины и сопротивления: каждая спираль реостата имеет определённое сопротивление. Поэтому плавно изменять силу тока с помощью такого прибора не получится.

 

 

Повторим формулы

Сопротивление проводника: R=ρ⋅lS

 

Из этой формулы можно выразить и другие величины:

 

l=R⋅Sρ, S=ρ⋅lR, ρ=R⋅Sl.

 

Удельное сопротивление и температурный коэффициент сопротивления металлов

Термисторы представляют собой чувствительные к колебаниям температуры сопротивления, часто используемые для автоматического обнаружения, измерения и контроля физической энергии. Важнейшее отличие термисторов от других материалов с переменным сопротивлением заключается в их исключительной чувствительности к сравнительно малым изменениям температуры. В противоположность металлам, имеющим небольшой температурный коэффициент сопротивления, термисторы обладают большим отрицательным температурным коэффициентом. Обычно термисторы выполняют в виде бусинок, дисков или шайб и стержней. Их изготовляют из смесей окислов различных металлов, таких, как марганец, никель, кобальт, медь, уран, железо, цинк, титан и магний, со связующими материалами. Окислы смешивают в определенных пропорциях, обеспечивающих получение требуемого удельного сопротивления и температурного коэффициента сопротивления. Полученным смесям придают нужную форму и спекают в контролируемых атмосферных и температурных условиях. Окончательный продукт представляет собой твердый керамический материал, который можно монтировать различными способами в зависимости от механических, температурных и электрических требований.  [c.359]
Средние значения удельного электрического сопротивления и температурного коэффициента электросопротивления металлов при 20 °С  [c.231]

Удельное сопротивление и температурный коэффициент сопротивления металлов  [c.115]

В случаях, когда металлы сильно отличаются друг от друга объемами своих атомов и температурами плавления при этом удельное сопротивление и температурный коэффициент удельного сопротивления линейно изменяются в зависимости от содержания примеси в пределах от О до 100%, как это видно на рис. 4-1, а, на котором показана зависимость от соотношения компонент удельного сопротивления и его температурного коэффициента для сплава хрома и висмута, образуюш,их в сплаве механическую смесь.  [c.248]

Так как для сплавов р обычно много больше р , то вплоть до высоких температур их удельное сопротивление меняется с температурой значительно слабее, чем у чистых металлов, и температурный коэффициент сопротивления сплавов, как правило, значительно ниже температурного коэффициента сопротивления чистых металлов.  [c.190]

Характер влияния металлической примеси на величину удельного сопротивления данного металла зависит от типа образуемого сплава. Различают три типа сплавов механическая смесь, твердый раствор и химическое соединение. В первом случае в сплаве содержатся кристаллы обоих металлов — кристаллы примеси механически смешаны с кристаллами основного металла. Такой сплав получается в случаях, когда металлы сильно отличаются друг от друга объемами своих атомов и температурами плавления при этом удельное сопротивление и температурный коэффициент удельного сопротивления линейно изменяются в зависимости от содержания примеси в пределах от О до 100%, как это видно на рис. 6-1,а, на котором показана зависимость удельного сопротивления и температурного коэффициента от пропорции алюминия и свинца, образующих в сплаве механическую смесь.  [c.246]


Чистые благородные металлы имеют низкое удельное электросопротивление и высокий температурный коэффициент. Температурный коэффициент электросопротивления значительно уменьшается в присутствии ничтожных количеств примесей, и поэтому величина его является критерием чистоты металла. Устойчивость электросопротивления и температурного коэффициента платины используется в термометрах сопротивления.  [c.397]

Материалы, из которых изготовляются термометры сопротивления, должны обладать большим температурным коэффициентом сопротивления, большим удельным сопротивлением, постоянством химических и физических свойств, а зависимость сопротивления металла от температуры должна выражаться плавной кривой. Предъявляемым требованиям удовлетворяют платина и медь, из которых изготовляют технические термометры сопротивления. Платиновые термометры сопротивления предназначаются для длительного измерения температуры в пределах от — 200 до 4-500° С, а медные —в пределах от — 50 до -МОО°С. Медные термометры сопротивления могут быть использованы для кратковременных измерений температуры до 150°С.  [c.57]

В процессе первичного нагрева до 1100 °С N1—В-покрытиям свойствен отрицательный температурный коэффициент сопротивления и только при повторном нагреве электрическое сопротивление, как у большинства металлов, начинает монотонно возрастать. После термической обработки удельное электрическое сопротивление осадков снижается.  [c.389]

Чтобы повысить величину удельного сопротивления проводников, применяют сплавы нескольких металлов. Установлено, что только сплавы с неупорядоченной структурой обладают повышенными значениями удельного сопротивления и малыми значениями температурного коэффициента сопротивления. Сплавами с неупорядоченной структурой называются такие, в кристаллической решетке которых нет правильного чередования атомов металлов, составляющих сплав. Эти сплавы составляют группу проводниковых материалов с большим удельным сопротивлением и малыми значениями температурного коэффициента удельного сопротивления. Все перечисленные группы проводников обладают высокой пластичностью, позволяющей получать провода диаметром до 0,01 мм и ленты толщиной 0,05—0,1 мм.  [c.100]

Метод электрического сопротивления может быть применен для определения величины удельного электрического сопротивления р и температурного коэффициента электрического сопротивления характеристики металлов и сплавов, применяемых в электротехнике, а также для изучения фазовых и структурных превращений, протекающих в металлах и сплавах в твердом состоянии. Например, при исследовании процесса отпуска стали получают кривую электрическое сопротивление — температура отпуска. Изменение электрического сопротивления, характеризуемое этой кривой, указывает на превращения, протекающие в стали при отпуске.  [c.25]

Значения удельного сопротивления, его температурного коэффициента и температурного коэффициента линейного расширения некоторых металлов  [c.31]

Кроме металлов для изготовления термометров сопротивления применяют также полупроводниковые материалы германий, окислы меди, марганца, кобальта, магния, титана и их смеси. Большинство полупроводниковых материалов обладает большим отрицательным температурным коэффициентом сопротивления и также очень большим удельным сопротивлением. Поэтому можно изготавливать очень малые по размерам чувствительные элементы термопреобразователей сопротивления, обладающих  [c.46]

Для практической термометрии интерес представляют переходные металлы, имеющие частично заполненные -уровни, а также з-уровни (символы з и соответствуют значениям орбитального квантового числа О и 2 см. [6]). Поскольку -электроны более локализованы, чем з-электроны, проводимость обусловлена главным образом последними. Однако вероятность рассеяния 3-электронов в -зону велика, поскольку плотность -состояний вблизи уровня Ферми высока (рис. 5.5), поэтому удельное сопротивление переходных металлов выще, чем у непереходных. Наличие -зоны влияет также на характер температурной зависимости. При высоких температурах величина кТ может быть уже не пренебрежимо мала по сравнению с расстоянием от уровня Ферми до верхней или нижней границы -зоны. Предположение, что поверхность Ферми четко разделяет занятые и незанятые состояния, перестает быть верным, и для параболической -зоны в формулу удельного сопротивления вводится поправочный коэффициент (1—5Р), где В — постоянная. Однако плотность состояний в -зоне вовсе не является гладкой функцией энергии (рис. 5.5), поэтому эффект будет осложнен изменением плотности состояний в пределах кТ от уровня Ферми. Отклонение температурной зависимости от линейной может быть как положительным, так и отрицательным.  [c.194]


Температурный коэффициент удельного сопротивления металлов. Число носителей заряда (концентрация свободных электронов) в металлическом проводнике при повышении температуры практически остается неизменным. Однако вследствие усилений колебаний узлов кристаллической решетки с ростом температуры появляется все больше и больше препятствий на пути направленного движения свободных электронов под действием электрического поля, т. е. уменьшается средняя длина свободного пробега электрона X, уменьшается подвижность электронов и, как следствие, уменьшается удельная проводимость металлов и возрастает удельное сопротивление (рис. 7-2). Иными словами, температурный коэффициент (см. стр. 39) удельного сопротивления металлов (кельвин в минус первой степени)  [c.192]

Опыт, накопленный при изучении проводимости металлов и сплавов, экспериментальная техника, созданная для исследования электроизоляционных материалов, служат базой для определения электрических свойств покрытий. Рассматриваются многие свойства удельное электрическое сопротивление, электрическая прочность , электрическая проводимость, контактное сопротивление между покрытием и основным металлом, диэлектрическая проницаемость,, температурный коэффициент электрического сопротивления. Что касается керамических покрытий, которые используются в качестве электроизоляционного материала, то основным их свойством следует считать электрическую прочность. За электрическую прочность часто принимают напряженность пробоя, отнесенную к усредненной толщине покрытия.  [c.85]

Удельное электрическое сопротивление кристалла графита вдоль спайности равно 0,4-iO ом-сл1 и имеет положительный температурный коэффициент, как большинство металлов. У графитовых блоков и порошков к объемному сопротивлению вещества присоединяется сопротивление контактов между кристаллитами и зернами. Поэтому сопротивление блоков из крупнокристаллического графита значительно больше и составляет (0,5 5,0) 10 ом-см, а для высокодисперсного графита (8-н20)-10 ом-см.  [c.405]

Материалы с разными типами связи имеют различные температурные коэффициенты электросопротивления-, у металлов он положителен, у материалов с ковалентным и ионным типом связи — отрицателен. При нагреве металлов концентрация носителей зарядов — электронов не увеличивается, а сопротивление их движению возрастает из-за увеличения амплитуд колебаний атомов. В материалах с ковалентной или ионной связью при нагреве концентрация носителей зарядов повышается настолько, что нейтрализуется влияние помех от увеличения колебаний атомов. По этой причине удельное электросопротивление таких материалов при нагреве снижается. Начиная с (0,8-0,9)Тпл концентрация носителей заряда становится большой, а сами материалы делаются проводящими.  [c.67]

Другие системы. Некоторые теллуриды и селениды исследовались также при стехиометрическом составе, однако полученные результаты недостаточно надежны (изучение концентрационной зависимости свойств существенно важнее, так как при этом можно избежать проблемы измерения свойств при точном стехиометрическом составе, поскольку данные для этого состава можно получить интерполяцией). Температурные коэффициенты у этих соединений обычно отрицательные в жидком состоянии и удельное сопротивление после плавления уменьшается, но проводимость в жидком состоянии достаточно высока. Такие же результаты получены для силицидов переходных металлов, у которых удельное сопротивление в жидком состоянии примерно равно 3-10 мком-см. Для некоторых сплавов имеются сообщения о скачкообразном изменении температурного коэффициента удельного сопротивления аь при температурах, находящихся вблизи точки плавления [70, 376, 377]. Ясно, что необходимо продолжить исследования, поскольку эти наблюдения говорят о возможности изменений в дискретной структуре жидких сплавов, выраженных, возможно, в форме фазовых изменений . Кажется, никто сильно не возражает против возможности нестабильности одной жидкой структуры по отношению к другой при некоторой критической температуре, хотя при высоких температурах (и, следовательно, высоких амплитудах атомных колебаний) структуры должны быть очень стабильными. Эти явления, возможно, связаны с изменением а К) из-за температуры, так как эта функция тоже влияет на температур-  [c.134]

Температурный коэффициент электрического сопротивления монокристалла графита положительный, как и для большинства металлов с преобладающей электронной проводимостью. Для порошков и блоков этот коэффициент отрицательный при не очень высоких температурах. Для составов, соот-ветствующих промышлен-/ 1— ным сортам графита, минимум электросопротивления обусловлен сложениеМ[ двух противоположно действующих факторов с одной стороны, электросопротивление кристаллитов графита с повышением температуры увеличивается, с другой — улучшается контакт между ними. При низких температурах преобладает второй фактор, при высоких — первый I]. Чем выше дисперсность материала, тем больше будет величина электросопротивления при низких температурах и тем глубже будет минимум электросопротивления. Выше 1000° С электросопротивление растет, как правило, пропорционально температуре. На рис, 9 представлены кривые изменения электросопротивления некоторых сортов графита [73]. Аналогичные значения приводят авторы работ [75 237, с. 74]. Для температур выше 1000° С удельное сопротивление можно рассчитать по формуле  [c.38]


Зависимость удельного сопротивления металлов от температуры. Введем понятие температурного коэффициента, которым будем пользоваться и в дальнейшем. Температурный коэффициент (ТК) какого-либо параметра г материала или радиоэлектронного компонента — это логарифмическая производная этого параметра по температуре  [c.15]

Как отмечалось, р металлов при повышении Г возрастает (см. рис. 1.1, 1.2 и 3.6). Следовательно, температурный коэффициент удельного сопротивления металлов  [c.15]

Полупроводники по удельному сопротивлению, которое при комнатной температуре составляет 10 — 10 Ом -м, занимают промежуточное положение между металлами и диэлектриками. Они обладают совокупностью специфических свойств, которые и выделяют их среди других веществ. В отличие от металлов полупроводники имеют в большом интервале температур отрицательный температурный коэффициент удельного сопротивления ТКр, т. е. положительный температурный коэффициент удельной проводимости ТКу (рис. И.1).  [c.47]

К сплавам высокого сопротивления относят сплавы, обладающие большим удельным сопротивлением (большим, чем серебро, медь и другие хорошие проводники) сплавы с малым температурным коэффициентом сплавы, не расплавляющиеся и ие окисляющиеся при высокой температуре. Сплавы высокого сопротивления условно делят на сплавы никелевые и медно-никелевые и жаростойкие металлы и сплавы.  [c.400]

Изменение сопротивления самого нагревателя в процессе окисления будет определяться соотношением степени уменьшения сечения нагревателя в результате перехода металла в оксиды и изменения удельного сопротивления и его температурного коэффициента. Наибольшее изменение сопротивления отмечается для тонких проволок с диаметром 1,0 мм и менее. Например, в трубчатых электронагревателях (ТЭН) сопротивление нагревателя при комнатной температуре в процессе эксплуатации может уменьшиться на 60—70%- Отношение же сопротивления нагревателя при температуре эксплуатации к сопротивлению при комнатной температуре увеличивается от 1,04—1,08 в начале эксплуатации до 1,7—1,8 в конце [10—12].  [c.21]

НИИ удельное сопротивление имеет максимум, а температурный коэффициент удельного сопротивления минимум, как это видно на рис. 6-1,6. Таким характером обладает сплав меди и никеля. При ограниченной растворимости одного металла в другом изменение удельного сопротивления сплава показано а рис. 6-1,в. При малом содержании примеси, не выходящем за пределы растворимости в основном металле, зависимость удельного сопротивления от содержания примеси линейная.  [c.286]

В основе электротехнических угольных материалов лежат графит и уголь — разновидности почти чистого углерода, являющегося полупроводником, вследствие чего графит и уголь имеют отрицательный температурный коэффициент удельного сопротивления, хотя по величине проводимости немногим уступают металлам и их сплавам. Важнейшими видами электротехнических угольных изделий являются 1) щетки для электрических машин 2) угольные электроды (для электрических печей, электролитических ванн и сварки) 3) осветительные угли 4) непроволочные сопротивления 5) микрофонные порошки, мембраны и другие детали техники связи 6) части гальванических- элементов 7) детали электровакуумных приборов (аноды, сетки).  [c.334]

В табл. 19.1 представлены значения удельного сопротивления и температурного коэффициента сопротивления чистых металлов, а также, в некоторых случаях, отношение удельного сопротивления при температуре жидкого гелия к удельному сопротивлению при нормальных условиях, р4.2°к/р273 к, характеризующее достигнутую степень чистоты материала. В тех случаях, когда для данного металла приводятся более подробные данные, соответствующее указание дается в первом столбце таблицы. Металлы в таблице расположены в порядке возрастания массового числа.  [c.304]

Существует класс полупроводниковых приборов, выполненных на основе смешанных окислов переходных металлов, которые известны под общим названием термисторов. Термин термистор происходит от слов термочувствительный резистор . Толчком к разработке термисторов послужила необходимость компенсировать изменение параметров электронных схем под влиянием колебаний температуры. Первые термисторы изготавливались на основе двуокиси урана ПОг, но затем в начале 30-х годов стали использовать шпинель MgTiOз. Оказалось, что удельное сопротивление MgTiOз и его температурный коэффициент сопротивления (ТКС) легко варьируются путем контролируемого восстановления в водороде и путем изменений концентрации MgO по сравнению со стехиометрической. Использовалась также окись меди СиО. Современные термисторы [60, 61] почти всегда представляют собой нестехиометрические смеси окислов и изготавливаются путем спекания микронных частиц компонентов в контролируемой атмосфере. В зависимости от того, в какой атмосфере происходит спекание (окислительной или восстановительной), может получиться, например, полупроводник п-типа на поверхности зерна, переходящий в полупроводник р-типа в глубине зерна, со всеми вытекающими отсюда последствиями для процессов проводимости. Помимо характера проводимости в отдельном зерне, на проводимость материала оказывают существенное влияние также процессы на границах между спеченными зернами. Высокочастотная дисперсия у термисторов, например, возникает вследствие того, что они представляют собой сложную структуру, образованную зонами плохой проводимости на границах зерен и зонами относительно высокой проводимости внутри зерен.  [c.243]

Серебро. Среди металлов серебро — наиболее низкоомный проводник величина р = 0,016 ом Температурный коэффициент сопротивления TKR = 3,6 10 /1 град. Температура плавления серебра 960° С. Серебро отличается небольшой твердостью оно является высокопластичным металлом, легко претерпевающим упругие деформации. Его окисление на воздухе при нормальной температуре протекает весьма медленно, поэтому его используют для покрытий проводников в высокочастотных элементах. При высоких частотах сопротивление посеребренного проводника может быть в десятки раз ниже, чем медного. При повышенных температурах (свыше 200° С) серебро на воздухе начинает окисляться. Если в воздухе присутствуют сернистые соединения, то на поверхности образуется слой сернистого серебра AgjS с высоким удельным сопротивлением. Для защиты серебряного покрытия от окисления и воздействия сернистых соединений в некоторых случаях, на него наносят слой лака или весьма тонкий слой (толщиной доли микрона) палладия. Из серебра выполняют электроды слюдяных и керамических конденсаторов проводниковые элементы схем, провода высокочастотных катушек и т. п. Серебро является компонентом различных сплавов и контактных материалов.  [c.274]

Медь. Вторым после серебра металлом с низким сопротивлением является медь. Для проводников используется электролитическая медь с содержанием Си 99,9% и кислорода 0,08%. Высокой вязкостью и пластичностью обладает бескислородная медь, содержащая кислорода не более 0,02%. Температура плавления меди 1084° С, температура рекристаллизации — около 270° С. При нагревании выше этой температуры резко снижается прочность и возрастает пластичность. На воздухе поверхность медного проводника быстро покрывается слоем закиси — окиси меди с высоким удельным сопротивлением. Высокочастотные медные токоведущие элементы защищают от окисления покрытием из серебра. Для обмоток маслонаполненных трансформаторов используют луженую медную проволоку. Техническая медная проволока диаметром от 0,1 до 12 мм выпускается твердая и мягкая, подвергаемая отжигу в печах без доступа воздуха. Мягкая проволока диаметром до 3 мм имеет временное сопротивление в среднем 0р = 27 /сГ/лл для твердой проволоки больше (Ор = 39 кГ мм% удельное сопротивление для твердой проволоки р = 0,018 ом -мм 1м, а для мягкой р = 0,0175 ом-мм м. Температурный коэффициент сопротивления меди TKR =4-45-10″ Ijapad. Твердую медь применяют для контактных проводэв, коллекторов и т. п. Во всех этих  [c.274]


Жаростойкими проводниковыми материалами являются сплавы на основе никеля, хрома и некоторых других компонентов. Жароупорность этих сплавов, т. е. их неокисляемость даже при высоких температурах, обусловлена образованием на их поверхности окисной пленки большой сплошности, исключающей доступ кислорода к сплаву. Основой жаростойких окисных пленок является окись хрома (СгаОз) и закись никеля (N10), которые не испаряются с поверхности сплава при высоких температурах. Жаростойкие проводниковые материалы на основе никеля, хрома и алюминия называются соответственно нихромами, фехралями и хромалями. Все они представляют собой твердые растворы металлов с неупорядоченной структурой, поэтому эги сплавы обладают большим удельным сопротивлением и малыми значениями температурного коэффициента сопротивления.  [c.105]

Наряду с малым удельным сопротивлением чистые металлы обладают хорошей пластичностью, т. е. могут вытягиваться в тонкую проволоку (до диаметра 0,01 мм), ленты (до толщины 0,01 мм) и прокатываться в фольгу толщиной менее 0,01 мм. Сплавы металлов обладают меньшей пластичностью по сравнению с чистыми металлами, они более упруги и имеют большую механическую прочность. Характерной особенностью всех металлических проводниковых, материалов является их электронная электропроводность. Удельное сопротивление всех металлических проводников увеличивается с ростом температуры и в зависимости от температурного коэффициента сопротивления, а также в результате механической обработки, вызывающей остаточную деформацию в металле. К холодной обработке (прокатка, волочение) приходится прибегать для получения проводниковых изделий с повышенным пределом прочности при разрыве, например при изготовлении проводов воздушных линий, троллейных проводов и т. д. Чтобы вернуть деформированным металлическ , проводникам прежнюю величину удельного сопротивления, их подвергают термической обработке — отжигу без доступа кислорода.  [c.225]

Ответ. 1) У металлов удельное сопротивление при нормальной температуре находится в пределах 10 —10- Ом-м см. рис. 2-1-1), а температурный коэффициент сопротивления положителен. В области температур, близких к комнатной, удельное сопротивление, как правило, пропорцнонально температуре, однако в диапазоне температур от 20 К л0 дебаевской удельное сопротивление большинства металлов становится пропорциональным Г . В противоположность металлам материалы с ионной проводимостью и все полупроводники имеют отрицательный температурный коэффициент сопротивления.  [c.345]

Металлические рубидий и цезий были получены в стеклянных ампулах ОХМЗ ГИРЕДМЕТ. Чистота металлов — 99,97%. В точке плавления как со стороны твердой фазы, так и со стороны жидкой не отмечено никаких аномалий в температурной зависимости удельного электросопротивления. Сопротивление в твердой фазе, а также в жидкой для рубидия до 110° С, а для цезия до 230° С линейно зависит от температуры. Температура плавления (начало плавления) для рубидия 38,8 + 0,1° С, а для цезия — 28,3 + +0,1° С. Отмечено переохлаждение рубидия до 38,4 °С, а цезия — до 28,1° С. Скачок сопротивления в точке плавления для рубидия рш/ртв = 1,562, а для цезия — 1,704. Температурный коэффициент сопротивления— (1/р)- йр1йТ) в точке плавления со стороны] твердой фазы для рубидия  [c.152]

Никель и изготовляемые из него термометры сопротивления. Основным досто-ннстБом никеля является то, что он обладает высоким температурным коэффициентом электрического сопротивления (сс == 6,66 1(Г °С ) и большим удельным сопротивлением (р 12,8 — 10 S Ом м). К числу недостатков никеля следует отнести значительную окисляемость при высоких температурах и большую зависимость температурного коэффициента сопротивления от степени чистоты металла. Зависимость сопротивления никеля от температуры имеет резко нелинейную характеристику (рис. 5-2-1).  [c.196]

В электро- и радиоаппаратостроении применяют материалы с высоким удельным электрическим сопротивлением р. Обычно это сплавы полностью однородных твердых растворов с высокой концентрацией или сплавы, основная масса которых состоит из таких растворов (поскольку р их выше, а температурный коэффициент электросопротивления значительно ниже, чем у исходных металлов).  [c.282]

Свойства проводников. К важнейшим параметрам, характеризующим свойства проводниковых материалов, относятся 1) удельная проводимость у или обратная ей величина — удельное сопроти13ление р, 2) температурный коэффициент удельного сопротивления ТКр или р, 3) коэффициент теплопроводности 4) контактная разность потенциалов и термоэлектродвижущая сила (термо-ЭДС), 5) работа выхода электронов из металла, 6) предел прочности при растяжении сГр и относительное удлинение перед разрывом А///.  [c.190]

В сплавах со структурой твердых растворов удельное электросопротивление при 20 °С в зависимости от состава, согласно правилу Н.С. Кур-накова, изменяется по нелинейной зависимости (рис. 18.5). Сплав приобретает максимальное значение р в большинстве случаев при концентрации элементов, равной 50 % (ат.). Видимо, в таком сплаве примесное рассеяние вследствие искажений кристаллической решетки и нарушения периодичности энергетических зон достигает максимального значения. В тех сплавах, в которых хотя бы один из элементов является переходным металлом, температурный коэффициент ар может принимать отрицательные значения, т.е. электрическое сопротивление при нагреве несколько уменьшается. В тех случаях, когда необходим материал с повышенным электрическим сопротивлением, следует использовать сплавы со структурой твердых растворов.  [c.573]

Пленки больщинства металлов (например, благородных металлов и неферромагнитных металлов переходной группы) толщиной в несколько сот ангстрем имеют удельное электросопротивление, величина которого изменяется с изменением температуры так же, как и у сплошных металлов. Однако пленки этих металлов толщиной в несколько ангстрем имеют большое удельное электросопротивление и большой отрицательный температурный коэффициент. Зависи.мость сопротивления этих пленок от те.мпературы в широком интервале температур описывается уравнением, характерным для примесных полупроводников. Энергия активации в сильной степени зависит от состава пленок и их толщины. Для пленок значительной толщины, но еще не настолько толстых, чтобы появились типичные металлические свойства, зависимость сопротивления от температуры оказывается более сложной. Характеристики этих пленок и воспроизводимость их свойств сильно зависят от способа приготовления пленки, от присутствия сорбируе.мых газов, а в некоторых случаях — от старения и отжига.  [c.181]

Зависимость удельного сопротивления сплава двух металлов, образующих друг с другом твердый раствор, от изменения содержания каждого из них в пределах от нуля до 100% представлена графически на фиг. 134 (верхний левый график). Кривая удельного сопротивления имеет максимум, соответствующий некоторому. определенному соотношению компонентов сплава, и, при уменьшении содержания каждого из них удельное сопротивление падает, приближаясь к соответствующим значениям р чистых металлов. Обычно при этом наблюдается определенная закономерность и для изменения ТКр относителыю высокими значениями температурного коэффициента удельного сопротивления обладают чистые металлы, а у сплавов ТКр меньше и даже может достигать небольших отрицательных значений (фиг. 134 — левый нижний график).  [c.269]

Температурный коэффициент удельного сопрот ивления у жидкого металла, как правило, меньше, чем у него же в твердом состоянии, как это следует из выражения (7-10) и табл. 7-4, однако у некоторых металлов он может быть и больше. Знак температурного коэ(] и-циента удельного сопротивления в большинстве случаев положителен.  [c.285]


Сопротивление меди и стали. Удельное сопротивление железа, алюминия, меди и других металлов


    Удельное сопротивление железа, алюминия и других проводников

    Передача электроэнергии на дальние расстояния требует заботиться о минимизации потерь, происходящих от преодоления током сопротивления проводников, составляющих электрическую линию. Разумеется, это не значит, что подобные потери, происходящие уже конкретно в цепях и устройствах потребления, не играют роли.

    Поэтому важно знать параметры всех используемых элементов и материалов. И не только электрические, но и механические. И иметь в распоряжении какие-то удобные справочные материалы, позволяющие сравнивать характеристики разных материалов и выбирать для проектирования и работы именно то, что будет оптимальным в конкретной ситуации.В линиях передачи энергии, где задачей ставится наиболее продуктивно, то есть с высоким КПД, довести энергию до потребителя, учитывается как экономика потерь, так и механика самих линий. От механики — то есть устройства и расположения проводников, изоляторов, опор, повышающих/понижающих трансформаторов, веса и прочности всех конструкций, включая провода, растянутые на больших расстояниях, а также от выбранных для выполнения каждого элемента конструкции материалов, зависит и конечная экономическая эффективность линии, ее работы и затрат на эксплуатацию. Кроме того, в линиях, передающих электроэнергию, более высоки требования на обеспечение безопасности как самих линий, так и всего окружающего, где они проходят. А это добавляет затрат как на обеспечение проводки электроэнергии, так и на дополнительный запас прочности всех конструкций.

    Для сравнения данные обычно приводятся к единому, сопоставимому виду. Зачастую к таким характеристикам добавляется эпитет «удельный», а сами значения рассматриваются на неких унифицированных по физическим параметрам эталонах. Например, удельное электрическое сопротивление — это сопротивление (ом) проводника, выполненного из какого-то металла (меди, алюминия, стали, вольфрама, золота), имеющего единичную длину и единичное сечение в используемой системе единиц измерения (обычно в СИ). Кроме того, оговаривается температура, так как при нагревании сопротивление проводников может вести себя по-разному. За основу берутся нормальные средние условия эксплуатации — при 20 градусах Цельсия. А там, где важны свойства при изменении параметров среды (температуры, давления), вводятся коэффициенты и составляются дополнительные таблицы и графики зависимостей.

    Виды удельного сопротивления

    Так как сопротивление бывает:

    • активное — или омическое, резистивное, — происходящее от затрат электроэнергии на нагревание проводника (металла) при прохождении в нем электрического тока, и
    • реактивное — емкостное или индуктивное, — которое происходит от неизбежных потерь на создание всякими изменениями тока, проходящего через проводник электрических полей, то и удельное сопротивление проводника бывает двух разновидностей:
  1. Удельное электрическое сопротивление постоянному току (имеющее резистивный характер) и
  2. Удельное электрическое сопротивление переменному току (имеющее реактивный характер).

Здесь удельное сопротивление 2 типа является величиной комплексной, оно состоит из двух компонент ТП — активной и реактивной, так как резистивное сопротивление существует всегда при прохождении тока, независимо от его характера, а реактивное бывает только при любом изменении тока в цепях. В цепях постоянного тока реактивное сопротивление возникает только при переходных процессах, которые связаны с включением тока (изменение тока от 0 до номинала) или выключением (перепад от номинала до 0). И их учитывают обычно только при проектировании защиты от перегрузок.

В цепях же переменного тока явления, связанные с реактивными сопротивлениями, гораздо более многообразны. Они зависят не только от собственно прохождения тока через некоторое сечение, но и от формы проводника, причем зависимость не является линейной.


Дело в том, что переменный ток наводит электрическое поле как вокруг проводника, по которому протекает, так и в самом проводнике. И от этого поля возникают вихревые токи, которые дают эффект «выталкивания» собственно основного движения зарядов, из глубины всего сечения проводника на его поверхность, так называемый «скин-эффект» (от skin — кожа). Получается, вихревые токи как бы «воруют» у проводника его сечение. Ток течет в некотором слое, близком к поверхности, остальная толщина проводника остается неиспользуемой, она не уменьшает его сопротивление, и увеличивать толщину проводников просто нет смысла. Особенно на больших частотах. Поэтому для переменного тока измеряют сопротивления в таких сечениях проводников, где все его сечение можно считать приповерхностным. Такой провод называется тонким, его толщина равна удвоенной глубине этого поверхностного слоя, куда вихревые токи и вытесняют текущий в проводнике полезный основной ток.


Разумеется, уменьшением толщины круглых в сечении проводов не исчерпывается эффективное проведение переменного тока. Проводник можно утончить, но при этом сделать его плоским в виде ленты, тогда сечение будет выше, чем у круглого провода, соответственно, и сопротивление ниже. Кроме того, простое увеличение площади поверхности даст эффект увеличения эффективного сечения. Того же можно добиться, используя многожильный провод вместо одножильного, к тому же, многожилка по гибкости превосходит одножилку, что часто тоже бывает ценно. С другой стороны, принимая во внимание скин-эффект в проводах, можно сделать провода композитными, выполнив сердцевину из металла, обладающего хорошими прочностными характеристиками, например, стали, но невысокими электрическими. При этом поверх стали делается алюминиевая оплетка, имеющая меньшее удельное сопротивление.


Кроме скин-эффекта на протекание переменного тока в проводниках влияет возбуждение вихревых токов в окружающих проводниках. Такие токи называются токами наводки, и они наводятся как в металлах, не играющих роль проводки (несущие элементы конструкций), так и в проводах всего проводящего комплекса — играющих роль проводов других фаз, нулевых, заземляющих.

Все перечисленные явления встречаются во всех конструкциях, связанных с электричеством, это еще более усиливает важность иметь в своем распоряжении сводные справочные сведения по самым разным материалам.

Удельное сопротивление для проводников измеряется очень чувствительными и точными приборами, так как для проводки и выбираются металлы, имеющие самое низкое сопротивление -порядка ом *10-6 на метр длины и кв. мм. сечения. Для измерения же удельного сопротивления изоляции нужны приборы, наоборот, имеющие диапазоны очень больших значений сопротивления — обычно это мегомы. Понятно, что проводники обязаны хорошо проводить, а изоляторы хорошо изолировать.

Таблица

Железо как проводник в электротехнике

Железо — самый распространенный в природе и технике металл (после водорода, который металлом тоже является). Он и самый дешевый, и имеет прекрасные прочностные характеристики, поэтому применяется повсюду как основа прочности различных конструкций.

В электротехнике в качестве проводника железо используется в виде стальных гибких проводов там, где нужна физическая прочность и гибкость, а нужное сопротивление может быть достигнуто за счет соответствующего сечения.

Имея таблицу удельных сопротивлений различных металлов и сплавов, можно посчитать сечения проводов, выполненных из разных проводников.

В качестве примера попробуем найти электрически эквивалентное сечение проводников из разных материалов: проволоки медной, вольфрамовой, никелиновой и железной. За исходную возьмем проволоку алюминиевую сечением 2,5 мм.

Нам нужно, чтобы на длине в 1 м сопротивление провода из всех этих металлов равнялось сопротивлению исходной. Сопротивление алюминия на 1 м длины и 2,5 мм сечения будет равно

, где R – сопротивление, ρ – удельное сопротивление металла из таблицы, S – площадь сечения, L – длина.

Подставив исходные значения, получим сопротивление метрового куска провода алюминия в омах.

После этого разрешим формулу относительно S

, будем подставлять значения из таблицы и получать площади сечений для разных металлов.

Так как удельное сопротивление в таблице измерено на проводе длиной в 1 м, в микроомах на 1 мм2 сечения, то у нас и получилось оно в микроомах. Чтобы получить его в омах, нужно умножить значение на 10-6. Но число ом с 6 нулями после запятой нам получать совсем не обязательно, так как конечный результат все равно находим в мм2.

Как видим, сопротивление железа достаточно большое, проволока получается толстая.


Но существуют материалы, у которых оно еще больше, например, никелин или константан.

Похожие статьи:

domelectrik.ru

Таблица удельного электрического сопротивления металлов и сплавов в электротехнике

Главная > у >


Удельное сопротивление металлов.
Удельное сопротивление сплавов.
Значения даны при температуре t = 20° C. Сопротивления сплавов зависят от их точного состава. comments powered by HyperComments

tab.wikimassa.org

Удельное электрическое сопротивление | Мир сварки

Удельное электрическое сопротивление материалов

Удельное электрическое сопротивление (удельное сопротивление) — способность вещества препятствовать прохождению электрического тока.

Единица измерения (СИ) — Ом·м; также измеряется в Ом·см и Ом·мм2/м.

Материал Температура, °С Удельное электрическоесопротивление, Ом·м
Металлы
Алюминий200,028·10-6
Бериллий200,036·10-6
Бронза фосфористая200,08·10-6
Ванадий200,196·10-6
Вольфрам200,055·10-6
Гафний200,322·10-6
Дюралюминий200,034·10-6
Железо200,097·10-6
Золото200,024·10-6
Иридий200,063·10-6
Кадмий200,076·10-6
Калий200,066·10-6
Кальций200,046·10-6
Кобальт200,097·10-6
Кремний270,58·10-4
Латунь200,075·10-6
Магний200,045·10-6
Марганец200,050·10-6
Медь200,017·10-6
Магний200,054·10-6
Молибден200,057·10-6
Натрий200,047·10-6
Никель200,073·10-6
Ниобий200,152·10-6
Олово200,113·10-6
Палладий200,107·10-6
Платина200,110·10-6
Родий200,047·10-6
Ртуть200,958·10-6
Свинец200,221·10-6
Серебро200,016·10-6
Сталь200,12·10-6
Тантал200,146·10-6
Титан200,54·10-6
Хром200,131·10-6
Цинк200,061·10-6
Цирконий200,45·10-6
Чугун200,65·10-6
Пластмассы
Гетинакс20109–1012
Капрон201010–1011
Лавсан201014–1016
Органическое стекло201011–1013
Пенопласт201011
Поливинилхлорид201010–1012
Полистирол201013–1015
Полиэтилен201015
Стеклотекстолит201011–1012
Текстолит20107–1010
Целлулоид20109
Эбонит201012–1014
Резины
Резина201011–1012
Жидкости
Масло трансформаторное201010–1013
Газы
Воздух01015–1018
Дерево
Древесина сухая20109–1010
Минералы
Кварц230109
Слюда201011–1015
Различные материалы
Стекло20109–1013
ЛИТЕРАТУРА
  • Альфа и омега. Краткий справочник / Таллин: Принтэст, 1991 – 448 с.
  • Справочник по элементарной физике / Н.Н. Кошкин, М.Г. Ширкевич. М., Наука. 1976. 256 с.
  • Справочник по сварке цветных металлов / С.М. Гуревич. Киев.: Наукова думка. 1990. 512 с.

weldworld.ru

Удельное сопротивление металлов, электролитов и веществ (Таблица)

Удельное сопротивление металлов и изоляторов

В справочной таблице даны значения удельного сопротивления р некоторых металлов и изоляторов при температуре 18-20° С, выраженные в ом·см. Величина р для металлов в сильной степени зависит от примесей, в таблице даны значения р для химически чистых металлов, для изоляторов даны приближенно. Металлы и изоляторы расположены в таблице в порядке возрастающих значений р.

Таблица удельное сопротивление металлов

Чистые металлы

104 ρ (ом·см)

Чистые металлы

104 ρ (ом·см)

Алюминий

Дюралюминий

Платинит 2)

Аргентан

Марганец

Манганин

Вольфрам

Константан

Молибден

Сплав Вуда 3)

Сплав Розе 4)

Палладий

Фехраль 6)

Таблица удельное сопротивление изоляторов

Изоляторы

Изоляторы

Дерево сухое

Целлулоид

Канифоль

Гетинакс

Кварц _|_ оси

Стекло натр

Полистирол

Стекло пирекс

Кварц || оси

Кварц плавленый

Удельное сопротивление чистых металлов при низких температурах

В таблице даны значения удельного сопротивления (в ом·см) некоторых чистых металлов при низких температурах (0°С).

Отношение сопротивлении Rt/Rq чистых металлов при температуре Т °К и 273° К.

В справочной таблице дано отношение Rt/Rq сопротивлений чистых металлов при температуре Т °К и 273° К.

Чистые металлы

Алюминий

Вольфрам

Молибден

Удельное сопротивление электролитов

В таблице даны значения удельного сопротивления электролитов в ом·см при температуре 18° С. Концентрация растворов с дана в процентах, которые определяют число граммов безводной соли или кислоты в 100 г раствора.

Источник информации: КРАТКИЙ ФИЗИКО-ТЕХНИЧЕСКИЙ СПРАВОЧНИК/ Том 1, — М.: 1960.

infotables.ru

Удельное электрическое сопротивление — сталь

Cтраница 1

Удельное электрическое сопротивление стали возрастает с ростом температуры, причем наибольшие изменения наблюдаются при нагреве до температуры точки Кюри. После точки Кюри величина удельного электросопротивления изменяется незначительно и при температурах выше 1000 С практически остается постоянной.  

Ввиду большого удельного электрического сопротивления стали эти iuKii создают НсОольшое замедление в спадании потока. В контакторах на 100 а время отпадания составляет 0 07 сек, а в контакторах 600 а-0 23 сек. В связи с особыми требованиями, предъявляемыми к контакторам серии КМВ, которые предназначены для включения и отключения электромагнитов приводов масляных выключателей, электромагнитный механизм у этих контакторов допускает регулировку напряжения срабатывания и напряжения отпускания за счет регулировки силы возвратной пружины и специальной отрывной пружины. Контакторы типа КМВ должны работать при глубокой посадке напряжения. Поэтому минимальное напряжение срабатывания у этих контакторов может спускаться до 65 % UH. Такое низкое напряжение срабатывания приводит к тому, что при номинальном напряжении через обмотку протекает ток, приводящий к повышенному нагреву катушки.  

Присадка кремния увеличивает удельное электрическое сопротивление стали почти пропорционально содержанию кремния и этим способствует уменьшению потерь на вихревые токи, возникающие в стали при ее работе в переменном магнитном поле.  

Присадка кремния увеличивает удельное электрическое сопротивление стали, что способствует уменьшению потерь на вихревые токи, но одновременно кремний ухудшает механические свойства стали, делает ее хрупкой.  

Ом — мм2 / м — удельное электрическое сопротивление стали.  

Для уменьшения вихревых токов применяются сердечники, выполненные из сортов стали с повышенным удельным электрическим сопротивлением стали, содержащие 0 5 — 4 8 % кремния.  

Для этого на массивный ротор из оптимального сплава СМ-19 был надет тонкий экран из магнитно-мягкой стали. Удельное электрическое сопротивление стали мало отличается от удельного сопротивления сплава, а цг стали примерно на порядок выше. Толщина экрана выбрана по глубине проникновения зубцовых гармоник первого порядка и равна йэ 0 8 мм. Для сравнения приведены добавочные потери, Вт, при базовом короткозамкнутом роторе и двухслойном роторе с массивным цилиндром из сплава СМ-19 и с медными торцевыми кольцами.  

Основным магнитопроводящим материалом является листовая легированная электротехническая сталь, содержащая от 2 до 5 % кремния. Присадка кремния увеличивает удельное электрическое сопротивление стали, в результате чего уменьшаются потери на вихревые токи, сталь становится устойчивой к окислению и старению, но делается более хрупкой. В последние годы широко используется холоднокатаная текстурованная сталь с более высокими магнитными свойствами в направлении проката. Для уменьшения потерь от вихревых токов сердечник магнитопровода выполняется в виде пакета, собранного из листов штампованной стали.  

Электротехническая сталь является низкоуглеродистой сталью. Для улучшения магнитных характеристик в нее вводят кремний, который вызывает повышение удельного электрического сопротивления стали. Это приводит к уменьшению потерь на вихревые токи.  

После механической обработки магнитопровод отжигают. Так как в создании замедления участвуют вихревые токи в стали, следует ориентироваться на величину удельного электрического сопротивления стали порядка Рс (Ю-15) 10 — 6 ом см. В притянутом положении якоря магнитная система достаточно сильно насыщена, поэтому начальная индукция в различных магнитных системах колеблется в очень незначительных пределах и составляет для стали марки Э Вн1 6 — 1 7 гл. Указанное значение индукции поддерживает напряженность поля в стали порядка Ян.  

Для изготовления магнитных систем (магнитопроводов) трансформаторов применяются специальные тонколистовые электротехнические стали, имеющие повышенное (до 5 %) содержание кремния. Кремний способствует обезуглероживанию стали, что приводит к увеличению магнитной проницаемости, снижает потери на гистерезис и увеличивает ее удельное электрическое сопротивление. Увеличение удельного электрического сопротивления стали позволяет уменьшить потери в ней от вихревых токов. Кроме того, кремний ослабляет старение стали (увеличение потерь в стали с течением времени), уменьшает ее магнитострикцию (изменение формы и размеров тела при намагничивании) и, следовательно, шум трансформаторов. В то же время наличие кремния в стали приводит к повышению ее хрупкости и затрудняет ее механическую обработку.  

Страницы:      1    2

www.ngpedia.ru

Удельное сопротивление | Викитроника вики

Удельное сопротивление — характеристика материала, определяющая его способность проводить электрический ток. Определяется как отношение электрического поля к плотности тока. В общем случае является тензором, однако для большинства материалов, не проявляющих анизотропных свойств, принимается скалярной величиной.

Обозначение — ρ

$ \vec E = \rho \vec j, $

$ \vec E $ — напряжённость электрического поля, $ \vec j $ — плотность тока.

Единица измерения СИ — ом-метр (ом·м, Ω·m).

Сопротивление цилиндра или призмы (между торцами) из материала длиной l, и сечением S по удельному сопротивлению определяется следующим образом:

$ R = \frac{\rho l}{S}. $

В технике применяется определение удельного сопротивления, как сопротивление проводника единичного сечения и единичной длины.

Удельное сопротивление некоторых материалов, используемых в электротехнике Править

Материал ρ при 300 К, Ом·м ТКС, К⁻¹
серебро 1,59·10⁻⁸ 4,10·10⁻³
медь 1,67·10⁻⁸ 4,33·10⁻³
золото 2,35·10⁻⁸ 3,98·10⁻³
алюминий 2,65·10⁻⁸ 4,29·10⁻³
вольфрам 5,65·10⁻⁸ 4,83·10⁻³
латунь 6,5·10⁻⁸ 1,5·10⁻³
никель 6,84·10⁻⁸ 6,75·10⁻³
железо (α) 9,7·10⁻⁸ 6,57·10⁻³
олово серое 1,01·10⁻⁷ 4,63·10⁻³
платина 1,06·10⁻⁷ 6,75·10⁻³
олово белое 1,1·10⁻⁷ 4,63·10⁻³
сталь 1,6·10⁻⁷ 3,3·10⁻³
свинец 2,06·10⁻⁷ 4,22·10⁻³
дюралюминий 4,0·10⁻⁷ 2,8·10⁻³
манганин 4,3·10⁻⁷ ±2·10⁻⁵
константан 5,0·10⁻⁷ ±3·10⁻⁵
ртуть 9,84·10⁻⁷ 9,9·10⁻⁴
нихром 80/20 1,05·10⁻⁶ 1,8·10⁻⁴
канталь А1 1,45·10⁻⁶ 3·10⁻⁵
углерод (алмаз, графит) 1,3·10⁻⁵
германий 4,6·10⁻¹
кремний 6,4·10²
этанол 3·10³
вода, дистиллированная 5·10³
эбонит 10⁸
бумага твёрдая 10¹⁰
трансформаторное масло 10¹¹
стекло обычное 5·10¹¹
поливинил 10¹²
фарфор 10¹²
древесина 10¹²
ПТФЭ (тефлон) >10¹³
резина 5·10¹³
стекло кварцевое 10¹⁴
бумага вощёная 10¹⁴
полистирол >10¹⁴
слюда 5·10¹⁴
парафин 10¹⁵
полиэтилен 3·10¹⁵
акриловая смола 10¹⁹

ru.electronics.wikia.com

Удельное электрическое сопротивление | формула, объемное, таблица

Удельное электрическое сопротивление является физической величиной, которая показывает, в какой степени материал может сопротивляться прохождению через него электрического тока. Некоторые люди могут перепутать данную характеристику с обыкновенным электрическим сопротивлением. Несмотря на схожесть понятий, разница между ними заключается в том, что удельное касается веществ, а второй термин относится исключительно к проводникам и зависит от материала их изготовления.

Обратной величиной данного материала является удельная электрическая проводимость. Чем выше этот параметр, тем лучше проходит ток по веществу. Соответственно, чем выше сопротивление, тем больше потерь предвидится на выходе.

Формула расчета и величина измерения

Рассматривая, в чем измеряется удельное электрическое сопротивление, также можно проследить связь с не удельным, так как для обозначения параметра используются единицы Ом·м. Сама величина обозначается как ρ. С таким значением можно определять сопротивление вещества в конкретном случае, исходя из его размеров. Эта единица измерения соответствует системе СИ, но могут встречаться и другие варианты. В технике периодически можно увидеть устаревшее обозначение Ом·мм2/м. Для перевода из этой системы в международного не потребуется использовать сложные формулы, так как 1 Ом·мм2/м равняется 10-6 Ом·м.

Формула удельного электрического сопротивления выглядит следующим образом:

R= (ρ·l)/S, где:

  • R – сопротивление проводника;
  • Ρ – удельное сопротивление материал;
  • l – длина проводника;
  • S – сечение проводника.

Зависимость от температуры

Удельное электрическое сопротивление зависит от температуры. Но все группы веществ проявляют себя по-разному при ее изменении. Это необходимо учитывать при расчете проводов, которые будут работать в определенных условиях. К примеру, на улице, где значения температуры зависят от времени года, необходимые материалы с меньшей подверженностью изменениям в диапазоне от -30 до +30 градусов Цельсия. Если же планируется применение в технике, которая будет работать в одних и тех же условиях, то здесь также нужно оптимизировать проводку под конкретные параметры. Материал всегда подбирается с учетом эксплуатации.

В номинальной таблице удельное электрическое сопротивление берется при температуре 0 градусов Цельсия. Повышение показателей данного параметра при нагреве материала обусловлено тем, что интенсивность передвижения атомов в веществе начинает возрастать. Носители электрических зарядов хаотично рассеиваются во всех направлениях, что приводит к созданию препятствий при передвижении частиц. Величина электрического потока снижается.

При уменьшении температуры условия прохождения тока становятся лучше. При достижении определенной температуры, которая для каждого металла будет отличаться, появляется сверхпроводимость, при которой рассматриваемая характеристика почти достигает нуля.

Отличия в параметрах порой достигают очень больших значений. Те материалы, которые обладают высокими показателями, могут использовать в качестве изоляторов. Они помогают защищать проводку от замыкания и ненамеренного контакта с человеком. Некоторые вещества вообще не применимы для электротехники, если у них высокое значение этого параметра. Этому могут мешать другие свойства. Например, удельная электрическая проводимость воды не будет иметь большого значения для данный сферы. Здесь приведены значения некоторых веществ с высокими показателями.

Материалы с высоким удельным сопротивлениемρ (Ом·м)
Бакелит1016
Бензол1015…1016
Бумага1015
Вода дистиллированная104
Вода морская0.3
Дерево сухое1012
Земля влажная102
Кварцевое стекло1016
Керосин1011
Мрамор108
Парафин1015
Парафиновое масло1014
Плексиглас1013
Полистирол1016
Полихлорвинил1013
Полиэтилен1012
Силиконовое масло1013
Слюда1014
Стекло1011
Трансформаторное масло1010
Фарфор1014
Шифер1014
Эбонит1016
Янтарь1018

Более активно в электротехнике применяются вещества с низкими показателями. Зачастую это металлы, которые служат проводниками. В них также наблюдается много различий. Чтобы узнать удельное электрическое сопротивление меди или других материалов, стоит посмотреть в справочную таблицу.

Материалы с низким удельным сопротивлениемρ (Ом·м)
Алюминий2.7·10-8
Вольфрам5.5·10-8
Графит8.0·10-6
Железо1.0·10-7
Золото2.2·10-8
Иридий4.74·10-8
Константан5.0·10-7
Литая сталь1.3·10-7
Магний4.4·10-8
Манганин4.3·10-7
Медь1.72·10-8
Молибден5.4·10-8
Нейзильбер3.3·10-7
Никель8.7·10-8
Нихром1.12·10-6
Олово1.2·10-7
Платина1.07·10-7
Ртуть9.6·10-7
Свинец2.08·10-7
Серебро1.6·10-8
Серый чугун1.0·10-6
Угольные щетки4.0·10-5
Цинк5.9·10-8
Никелин0,4·10-6

Удельное объемное электрическое сопротивление

Данный параметр характеризует возможность пропускать ток через объем вещества. Для измерения необходимо приложить потенциал напряжения с разных сторон материала, изделие из которого будет включено в электрическую цепь. На него подается ток с номинальными параметрами. После прохождения измеряются данные на выходе.

Использование в электротехнике

Изменение параметра при разных температурах широко применяется в электротехнике. Наиболее простым примером является лампа накаливания, где используется нихромовая нить. При нагревании она начинает светиться. При прохождении через нее тока она начинает нагреваться. С ростом нагрева возрастает и сопротивление. Соответственно, ограничивается первоначальный ток, который нужен был для получения освещения. Нихромовая спираль, используя тот же принцип, может стать регулятором на различных аппаратах.

Широкое применение коснулось и благородных металлов, которые обладают подходящими характеристиками для электротехники. Для ответственных схем, которым требуется быстродействие, подбираются серебряные контакты. Они обладают высокой стоимостью, но с учетом относительно небольшого количества материалов их применение вполне оправданно. Медь уступает серебру по проводимости, но обладает более доступной ценой, благодаря чему ее чаще используют для создания проводов.

В условиях, где можно использовать предельно низкие температуры, применяются сверхпроводники. Для комнатной температуры и уличной эксплуатации они не всегда уместны, так как при повышении температуры их проводимость начнет падать, поэтому для таких условий лидерами остаются алюминий, медь и серебро.

На практике учитывается много параметров и этот является одним из наиболее важных. Все расчеты проводятся еще на стадии проектирования, для чего и используются справочные материалы.

Удельное сопротивление металлов является мерой их свойства противодействовать прохождению электрического тока. Эта величина выражается в Ом-метр (Ом⋅м). Символ, обозначающий удельное сопротивление, является греческая буква ρ (ро). Высокое удельное сопротивление означает, что материал плохо проводит электрический заряд.

Удельное сопротивление

Удельное электрическое сопротивление определяется как отношение между напряженностью электрического поля внутри металла к плотности тока в нем:

где:
ρ — удельное сопротивление металла (Ом⋅м),
Е — напряженность электрического поля (В/м),
J — величина плотности электрического тока в металле (А/м2)

Если напряженность электрического поля (Е) в металле очень большая, а плотность тока (J) очень маленькая, это означает, что металл имеет высокое удельное сопротивление.

Обратной величиной удельного сопротивления является удельная электропроводность, указывающая, насколько хорошо материал проводит электрический ток:

σ — проводимость материала, выраженная в сименс на метр (См/м).

Электрическое сопротивление

Электрическое сопротивление, одно из составляющих , выражается в омах (Ом). Следует заметить, что электрическое сопротивление и удельное сопротивление — это не одно и то же. Удельное сопротивление является свойством материала, в то время как электрическое сопротивление — это свойство объекта.

Электрическое сопротивление резистора определяется сочетанием формы и удельным сопротивлением материала, из которого он сделан.

Например, проволочный , изготовленный из длинной и тонкой проволоки имеет большее сопротивление, нежели резистор, сделанный из короткой и толстой проволоки того же металла.

В тоже время проволочный резистор, изготовленный из материала с высоким удельным сопротивлением, обладает большим электрическим сопротивлением, чем резистор, сделанный из материала с низким удельным сопротивлением. И все это не смотря на то, что оба резистора сделаны из проволоки одинаковой длины и диаметра.

В качестве наглядности можно провести аналогию с гидравлической системой, где вода прокачивается через трубы.

  • Чем длиннее и тоньше труба, тем больше будет оказано сопротивление воде.
  • Труба, заполненная песком, будет больше оказывать сопротивление воде, нежели труба без песка

Сопротивление провода

Величина сопротивления провода зависит от трех параметров: удельного сопротивления металла, длины и диаметра самого провода. Формула для расчета сопротивления провода:

Где:
R — сопротивление провода (Ом)
ρ — удельное сопротивление металла (Ом.m)
L — длина провода (м)
А — площадь поперечного сечения провода (м2)

В качестве примера рассмотрим проволочный резистор из нихрома с удельным сопротивлением 1.10×10-6 Ом.м. Проволока имеет длину 1500 мм и диаметр 0,5 мм. На основе этих трех параметров рассчитаем сопротивление провода из нихрома:

R=1,1*10 -6 *(1,5/0,000000196) = 8,4 Ом

Нихром и константан часто используют в качестве материала для сопротивлений. Ниже в таблице вы можете посмотреть удельное сопротивление некоторых наиболее часто используемых металлов.

Поверхностное сопротивление

Величина поверхностного сопротивления рассчитывается таким же образом, как и сопротивление провода. В данном случае площадь сечения можно представить в виде произведения w и t:


Для некоторых материалов, таких как тонкие пленки, соотношение между удельным сопротивлением и толщиной пленки называется поверхностное сопротивление слоя RS:

где RS измеряется в омах. При данном расчете толщина пленки должна быть постоянной.

Часто производители резисторов для увеличения сопротивления вырезают в пленке дорожки, чтобы увеличить путь для электрического тока.

Свойства резистивных материалов

Удельное сопротивление металла зависит от температуры. Их значения приводится, как правило, для комнатной температуры (20°С). Изменение удельного сопротивления в результате изменения температуры характеризуется температурным коэффициентом.

Например, в термисторах (терморезисторах) это свойство используется для измерения температуры. С другой стороны, в точной электронике, это довольно нежелательный эффект.
Металлопленочные резисторы имеют отличные свойства температурной стабильности. Это достигается не только за счет низкого удельного сопротивления материала, но и за счет механической конструкции самого резистора.

Много различных материалов и сплавов используются в производстве резисторов. Нихром (сплав никеля и хрома), из-за его высокого удельного сопротивления и устойчивости к окислению при высоких температурах, часто используют в качестве материала для изготовления проволочных резисторов. Недостатком его является то, что его невозможно паять. Константан, еще один популярный материал, легко паяется и имеет более низкий температурный коэффициент.

Электрическое сопротивление физическая величина, которая показывает, какое препятствие создается току при его прохождении по проводнику . Единицами измерения служат Омы, в честь Георга Ома. В своем законе он вывел формулу для нахождения сопротивления, которая приведена ниже.

Рассмотрим сопротивление проводников на примере металлов. Металлы имеют внутреннее строение в виде кристаллической решетки. Эта решетка имеет строгую упорядоченность, а её узлами являются положительно заряженные ионы. Носителями заряда в металле выступают “свободные” электроны, которые не принадлежат определенному атому, а хаотично перемещаются между узлами решетки. Из квантовой физики известно, что движение электронов в металле это распространение электромагнитной волны в твердом теле. То есть электрон в проводнике движется со скоростью света (практически), и доказано, что он проявляет свойства не только как частица, но еще и как волна. А сопротивление металла возникает в результате рассеяния электромагнитных волн (то есть электронов) на тепловых колебаниях решетки и её дефектах. При столкновении электронов с узлами кристаллической решетки часть энергии передается узлам, вследствие чего выделяется энергия. Эту энергию можно вычислить при постоянном токе , благодаря закону Джоуля-Ленца – Q=I 2 Rt. Как видите чем больше сопротивление, тем больше энергии выделяется.

Удельное сопротивление

Существует такое важное понятие как удельное сопротивление, это тоже самое сопротивление, только в единице длины. У каждого металла оно свое, например у меди оно равно 0,0175 Ом*мм2/м, у алюминия 0,0271 Ом*мм2/м. Это значит, брусок из меди длиной 1 м и площадью поперечного сечения 1 мм2 будет иметь сопротивление 0,0175 Ом, а такой же брусок, но из алюминия будет иметь сопротивление 0,0271 Ом. Выходит что электропроводность меди выше чем у алюминия. У каждого металла удельное сопротивление свое, а рассчитать сопротивление всего проводника можно по формуле

где p – удельное сопротивление металла, l – длина проводника, s – площадь поперечного сечения.

Значения удельных сопротивлений приведены в таблице удельных сопротивлений металлов (20°C)

Вещество

p , Ом*мм 2 /2

α,10 -3 1/K

Алюминий

0.0271

Вольфрам

0.055

Железо

0.098

Золото

0.023

Латунь

0.025-0.06

Манганин

0.42-0.48

0,002-0,05

Медь

0.0175

Никель

Константан

0.44-0.52

0.02

Нихром

0.15

Серебро

0.016

Цинк

0.059

Кроме удельного сопротивления в таблице есть значения ТКС, об этом коэффициенте чуть позже.

Зависимость удельного сопротивления от деформаций


При холодной обработке металлов давлением, металл испытывает пластическую деформацию. При пластической деформации кристаллическая решетка искажается, количество дефектов становится больше. С увеличением дефектов кристаллической решетки, сопротивление течению электронов по проводнику растет, следовательно, удельное сопротивление металла увеличивается. К примеру, проволоку изготавливают методом протяжки, это значит, что металл испытывает пластическую деформацию, в результате чего, удельное сопротивление растет. На практике для уменьшения сопротивления применяют рекристаллизационный отжиг, это сложный технологический процесс, после которого кристаллическая решетка как бы, “расправляется” и количество дефектов уменьшается, следовательно, и сопротивление металла тоже.

При растяжении или сжатии, металл испытывает упругую деформацию. При упругой деформации вызванной растяжением, амплитуды тепловых колебаний узлов кристаллической решетки увеличиваются, следовательно, электроны испытывают большие затруднения, и в связи с этим, увеличивается удельное сопротивление. При упругой деформации вызванной сжатием, амплитуды тепловых колебаний узлов уменьшаются, следовательно, электронам проще двигаться, и удельное сопротивление уменьшается.

Влияние температуры на удельное сопротивление

Как мы уже выяснили выше, причиной сопротивления в металле являются узлы кристаллической решетки и их колебания. Так вот, при увеличении температуры, тепловые колебания узлов увеличиваются, а значит, удельное сопротивление также увеличивается. Существует такая величина как температурный коэффициент сопротивления (ТКС), который показывает насколько увеличивается, или уменьшается удельное сопротивление металла при нагреве или охлаждении. Например, температурный коэффициент меди при 20 градусах по цельсию равен 4.1 · 10 − 3 1/градус. Это означает что при нагреве, к примеру, медной проволоки на 1 градус цельсия, её удельное сопротивление увеличится на 4.1 · 10 − 3 Ом. Удельное сопротивление при изменении температуры можно вычислить по формуле

где r это удельное сопротивление после нагрева, r 0 – удельное сопротивление до нагрева, a – температурный коэффициент сопротивления, t 2 – температура до нагрева, t 1 — температура после нагрева.

Подставив наши значения, мы получим: r=0,0175*(1+0.0041*(154-20))=0,0271 Ом*мм 2 /м. Как видите наш брусок из меди длиной 1 м и площадью поперечного сечения 1 мм 2 , после нагрева до 154 градусов, имел бы сопротивление, как у такого же бруска, только из алюминия и при температуре равной 20 градусов цельсия.

Свойство изменения сопротивления при изменении температуры, используется в термометрах сопротивления. Эти приборы могут измерять температуру основываясь на показаниях сопротивления. У термометров сопротивления высокая точность измерений, но малые диапазоны температур.

На практике, свойства проводников препятствовать прохождению тока используются очень широко. Примером может служить лампа накаливания, где нить из вольфрама, нагревается за счет высокого сопротивления металла, большой длины и узкого сечения. Или любой нагревательный прибор, где спираль разогревается благодаря высокому сопротивлению. В электротехнике, элемент главным свойством которого является сопротивление, называется – резистор . Резистор применяется практически в любой электрической схеме.

Уде́льное электри́ческое сопротивле́ние , или просто удельное сопротивление вещества — физическая величина, характеризующая способность вещества препятствовать прохождению электрического тока .

Удельное сопротивление обозначается греческой буквой ρ . Величина, обратная удельному сопротивлению, называется удельной проводимостью (удельной электропроводностью). В отличие от электрического сопротивления , являющегося свойством проводника и зависящего от его материала, формы и размеров, удельное электрическое сопротивление является свойством только вещества .

Электрическое сопротивление однородного проводника с удельным сопротивлением ρ , длиной l и площадью поперечного сечения S может быть рассчитано по формуле R = ρ ⋅ l S {\displaystyle R={\frac {\rho \cdot l}{S}}} (при этом предполагается, что ни площадь, ни форма поперечного сечения не меняются вдоль проводника). Соответственно, для ρ выполняется ρ = R ⋅ S l . {\displaystyle \rho ={\frac {R\cdot S}{l}}.}

Из последней формулы следует: физический смысл удельного сопротивления вещества заключается в том, что оно представляет собой сопротивление изготовленного из этого вещества однородного проводника единичной длины и с единичной площадью поперечного сечения.

Энциклопедичный YouTube

  • 1 / 5

    Единица измерения удельного сопротивления в Международной системе единиц (СИ) — Ом · . Из соотношения ρ = R ⋅ S l {\displaystyle \rho ={\frac {R\cdot S}{l}}} следует, что единица измерения удельного сопротивления в системе СИ равна такому удельному сопротивлению вещества, при котором однородный проводник длиной 1 м с площадью поперечного сечения 1 м² , изготовленный из этого вещества, имеет сопротивление, равное 1 Ом . Соответственно, удельное сопротивление произвольного вещества, выраженное в единицах СИ, численно равно сопротивлению участка электрической цепи, выполненного из данного вещества, длиной 1 м и площадью поперечного сечения 1 м² .

    В технике также применяется устаревшая внесистемная единица Ом·мм²/м, равная 10 −6 от 1 Ом·м . Данная единица равна такому удельному сопротивлению вещества, при котором однородный проводник длиной 1 м с площадью поперечного сечения 1 мм² , изготовленный из этого вещества, имеет сопротивление, равное 1 Ом . Соответственно, удельное сопротивление какого-либо вещества, выраженное в этих единицах, численно равно сопротивлению участка электрической цепи, выполненного из данного вещества, длиной 1 м и площадью поперечного сечения 1 мм² .

    Обобщение понятия удельного сопротивления

    Удельное сопротивление можно определить также для неоднородного материала, свойства которого меняются от точки к точке. В этом случае оно является не константой, а скалярной функцией координат — коэффициентом, связывающим напряжённость электрического поля E → (r →) {\displaystyle {\vec {E}}({\vec {r}})} и плотность тока J → (r →) {\displaystyle {\vec {J}}({\vec {r}})} в данной точке r → {\displaystyle {\vec {r}}} .{3}\rho _{ij}({\vec {r}})J_{j}({\vec {r}}).}

    В анизотропном, но однородном веществе тензор ρ i j {\displaystyle \rho _{ij}} от координат не зависит.

    Тензор ρ i j {\displaystyle \rho _{ij}} симметричен , то есть для любых i {\displaystyle i} и j {\displaystyle j} выполняется ρ i j = ρ j i {\displaystyle \rho _{ij}=\rho _{ji}} .

    Как и для всякого симметричного тензора, для ρ i j {\displaystyle \rho _{ij}} можно выбрать ортогональную систему декартовых координат, в которых матрица ρ i j {\displaystyle \rho _{ij}} становится диагональной , то есть приобретает вид, при котором из девяти компонент ρ i j {\displaystyle \rho _{ij}} отличными от нуля являются лишь три: ρ 11 {\displaystyle \rho _{11}} , ρ 22 {\displaystyle \rho _{22}} и ρ 33 {\displaystyle \rho _{33}} . В этом случае, обозначив ρ i i {\displaystyle \rho _{ii}} как , вместо предыдущей формулы получаем более простую

    E i = ρ i J i .{3}\sigma _{ij}({\vec {r}})E_{j}({\vec {r}}).}

    Из этого равенства и приведённого ранее соотношения для E i (r →) {\displaystyle E_{i}({\vec {r}})} следует, что тензор удельного сопротивления является обратным тензору удельной проводимости. С учётом этого для компонент тензора удельного сопротивления выполняется:

    ρ 11 = 1 det (σ) [ σ 22 σ 33 − σ 23 σ 32 ] , {\displaystyle \rho _{11}={\frac {1}{\det(\sigma)}}[\sigma _{22}\sigma _{33}-\sigma _{23}\sigma _{32}],} ρ 12 = 1 det (σ) [ σ 33 σ 12 − σ 13 σ 32 ] , {\displaystyle \rho _{12}={\frac {1}{\det(\sigma)}}[\sigma _{33}\sigma _{12}-\sigma _{13}\sigma _{32}],}

    где det (σ) {\displaystyle \det(\sigma)} — определитель матрицы , составленной из компонент тензора σ i j {\displaystyle \sigma _{ij}} . Остальные компоненты тензора удельного сопротивления получаются из приведённых уравнений в результате циклической перестановки индексов 1 , 2 и 3 .

    Удельное электрическое сопротивление некоторых веществ

    Металлические монокристаллы

    В таблице приведены главные значения тензора удельного сопротивления монокристаллов при температуре 20 °C .

    Кристаллρ 1 =ρ 2 , 10 −8 Ом·мρ 3 , 10 −8 Ом·м
    Олово9,914,3
    Висмут109138
    Кадмий6,88,3
    Цинк5,916,13
    • Константан (58,8 Cu, 40 Ni, 1,2 Mn)
    • Манганин (85 Cu, 12 Mn, 3 Ni)
    • Нейзильбер (65 Cu, 20 Zn, 15 Ni)
    • Никелин (54 Cu, 20 Zn, 26 Ni)
    • Нихром (67,5 Ni, 15 Cr, 16 Fe, 1,5 Mn)
    • Реонат (84Cu, 12Mn, 4 Zn)
    • Фехраль (80 Fe, 14 Cr, 6 Al)

    Удельное сопротивление нихрома

    Каждое тело, через которое пропускается электрический ток, автоматически оказывает ему определенное сопротивление. Свойство проводника противостоять электрическому току принято называть электрическим сопротивлением.

    Рассмотрим электронную теорию данного явления. При движении по проводнику свободные электроны постоянно встречают на своем пути другие электроны и атомы. Взаимодействуя с ними, свободный электрон теряет часть своего заряда. Таким образом, электроны сталкиваются с сопротивлением со стороны материала проводника. Каждое тело имеет свою атомную структуру, которая оказывает электрическому току разное сопротивление. Единицей сопротивления принято считать Ом. Обозначается сопротивление материалов — R или r.

    Чем меньше сопротивление проводника, тем легче электрическому току пройти через это тело. И наоборот: чем выше сопротивление, тем хуже тело проводит электрический ток.

    Сопротивление каждого отдельно взятого проводника зависит от свойств материала, из которого он изготовлен. Для точной характеристики электрического сопротивления того или иного материала было введено понятие — удельное сопротивление (нихрома, алюминия и т. д.). Удельным считается сопротивление проводника длиной до 1 м, сечение которого — 1 кв. мм. Этот показатель обозначается буквой p. Каждый материал, использующийся в производстве проводника, обладает своим удельным сопротивлением. Для примера рассмотрим удельное сопротивление нихрома и фехрали (более 3 мм):

    • Х15Н60 — 1.13 Ом*мм/м
    • Х23Ю5Т — 1.39 Ом*мм/м
    • Х20Н80 — 1.12 Ом*мм/м
    • ХН70Ю — 1.30 Ом*мм/м
    • ХН20ЮС — 1.02 Ом*мм/м

    Удельное сопротивление нихрома, фехрали указывает на основную сферу их применения: изготовление аппаратов теплового действия, бытовых приборов и электронагревательных элементов промышленных печей.

    Поскольку нихром и фехраль преимущественно используются в производстве нагревательных элементов, то самая распространенная продукция — нихромовая нить, лента, полоса Х15Н60 и Х20Н80, а также фехралевая проволока Х23Ю5Т.

Удельное сопротивление меди, таблица

Использование проводников из чистой меди и медных сплавов востребовано в различных отраслях промышленности. Материал имеет низкое удельное сопротивление, по данному параметру выделяется среди ряда других металлов. При организации протяженных кабельных трасс удельное сопротивление необходимо учитывать, так как потери на выходе могут быть значительными при передаче напряжения на большое расстояние.

Почему низкое удельное сопротивление – основная причина применения меди

С точки зрения физики, удельное сопротивление меди и других материалов показывает способность вещества препятствовать прохождению электрического тока, уровень потерь на единицу длины проводника. По сравнению с другими металлами, медь обладает низким удельным сопротивлением в 0,017, по данному показателю уступает только серебру. Благодаря подобным характеристикам медь является востребованным на рынке проводником:

  • низкое удельное сопротивление гарантирует минимальный уровень потерь при прохождении электрического тока;
  • внешние климатические нагрузки не оказывают значительного воздействия на технические характеристики металла;
  • расчет сечения кабеля выполнить намного проще за счет минимальных потерь между входом и выходом;
  • низкое сопротивление позволяет использовать для прокладки силовых, контрольных, специальных кабелей более тонкие проводники.

Стандартный медный провод поставляется в нескольких вариантах исполнения, отличается содержанием примесей, толщиной, характеристиками. При этом сопротивление меди может незначительно изменяться в зависимости от внешних условий.

Как рассчитать и от чего зависит сопротивление меди

Использование медного кабеля недостаточной толщины ведет к перегреву проводки, выходу оборудования из строя. При выборе излишне толстых проводов увеличиваются затраты. Поэтому важно знать сопротивление меди. Получить данную информацию можно несколькими способами:

  • наиболее простой вариант – получить подобную информацию из приведенной ниже таблицы. Для большинства случаев такой способ является рабочим, позволяет получить данные для проводников в стандартных условиях измерения;
  • более сложный вариант – расчет сопротивления меди по формуле. Для получения точных значений необходимо длину и сечение провода. Исходя их входных данных можно рассчитать сопротивление проводника;
  • еще один способ достаточно трудоемкий, предполагает использование специального прибора. С помощью омметра проводится измерение электрического сопротивления проводника с помощью подачи постоянного напряжения.

В зависимости от текущих условий измерения удельное сопротивление меди может меняться. При наличии льда в изоляции, непроводящего ток, сопротивление проводника увеличивается. Также параметры напрямую зависят от типа кабеля. Стандартные значения удельного сопротивления приведены в таблице:

Марка Медь О2 P Способ получения, основные примеси
М00к 99.98 0.01 Продукт электролитического рафинирования, заключительная стадия переработки медной руды.
М0к 99.97 0.015 0.001
М1к 99.95 0.02 0.002
М2к 99.93 0.03 0.002
М00 99.99 0.001 0.0003 Переплавка катодов в вакууме, инертной или восстановительной атмосфере. Уменьшает содержание кислорода.
М0 99.97 0.001 0.002
М1 99.95 0.003 0.002
М00 99.96 0.03 0.0005 Переплавка катодов в обычной атмосфере. Повышенное содержание кислорода. Отсутствие фосфора
М0 99.93 0.04
М1 99.9 0.05
М2 99.7 0.07 Переплавка лома. Повышенное содержание кислорода, фосфора нет
М3 99.5 0.08
М1ф 99.9 0.012 — 0.04 Переплавка катодов и лома меди с раскислением фосфором. Уменьшает содержание кислорода, но приводит к повышенному содержанию фосфора
М1р 99.9 0.01 0.002 — 0.01
М2р 99.7 0.01 0.005 — 0.06
М3р 99.5 0.01 0.005 — 0.06

Таблица удельного электрического сопротивления и проводимости

В этой таблице представлены удельное электрическое сопротивление и электропроводность нескольких материалов.

Удельное электрическое сопротивление, обозначаемое греческой буквой ρ (ро), является мерой того, насколько сильно материал противостоит прохождению электрического тока. Чем ниже удельное сопротивление, тем легче материал пропускает электрический заряд.

Электропроводность — это величина, обратная удельному сопротивлению.Электропроводность — это мера того, насколько хорошо материал проводит электрический ток. Электропроводность может быть представлена ​​греческой буквой σ (сигма), κ (каппа) или γ (гамма).

Таблица удельного сопротивления и проводимости при 20 ° C

Материал ρ (Ом • м) при 20 ° C
Удельное сопротивление
σ (См / м) при 20 ° C
Электропроводность
Серебро 1.59 × 10 −8 6,30 × 10 7
Медь 1,68 × 10 −8 5,96 × 10 7
Медь отожженная 1,72 × 10 −8 5,80 × 10 7
Золото 2,44 × 10 −8 4,10 × 10 7
Алюминий 2,82 × 10 −8 3,5 × 10 7
Кальций 3.36 × 10 −8 2,98 × 10 7
Вольфрам 5,60 × 10 −8 1,79 × 10 7
цинк 5,90 × 10 −8 1,69 × 10 7
Никель 6,99 × 10 −8 1,43 × 10 7
Литий 9,28 × 10 −8 1,08 × 10 7
Утюг 1.0 × 10 −7 1,00 × 10 7
Платина 1,06 × 10 −7 9,43 × 10 6
Олово 1,09 × 10 −7 9,17 × 10 6
Углеродистая сталь (10 10 ) 1,43 × 10 −7
Свинец 2,2 × 10 −7 4,55 × 10 6
Титан 4.20 × 10 −7 2,38 × 10 6
Электротехническая сталь с ориентированной зернистостью 4,60 × 10 −7 2,17 × 10 6
Манганин 4,82 × 10 −7 2,07 × 10 6
Константан 4,9 × 10 −7 2,04 × 10 6
Нержавеющая сталь 6,9 × 10 −7 1.45 × 10 6
Меркурий 9,8 × 10 −7 1,02 × 10 6
нихром 1,10 × 10 −6 9,09 × 10 5
GaAs 5 × 10 −7 до 10 × 10 −3 5 × 10 −8 до 10 3
Углерод (аморфный) 5 × 10 −4 до 8 × 10 −4 1.От 25 до 2 × 10 3
Углерод (графит) 2,5 × 10 −6 до 5,0 × 10 −6 // базисная плоскость
3,0 × 10 −3 ⊥ базальная плоскость
от 2 до 3 × 10 5 // базисная плоскость
3,3 × 10 2 ⊥ базальная плоскость
Карбон (алмаз) 1 × 10 12 ~ 10 −13
Германий 4,6 × 10 -1 2.17
Морская вода 2 × 10 -1 4,8
Питьевая вода 2 × 10 1 до 2 × 10 3 5 × 10 −4 до 5 × 10 -2
Кремний 6,40 × 10 2 1,56 × 10 −3
Дерево (влажное) 1 × 10 3 до 4 10 −4 до 10 -3
Деионизированная вода 1.8 × 10 5 5,5 × 10 −6
Стекло 10 × 10 10 до 10 × 10 14 10 −11 до 10 −15
Твердая резина 1 × 10 13 10 −14
Древесина (сушка в духовке) 1 × 10 14 до 16 10 −16 до 10 -14
сера 1 × 10 15 10 −16
Воздух 1.3 × 10 16 до 3,3 × 10 16 3 × 10 −15 до 8 × 10 −15
Парафиновый воск 1 × 10 17 10 −18
Плавленый кварц 7,5 × 10 17 1,3 × 10 −18
ПЭТ 10 × 10 20 10 −21
тефлон 10 × 10 22 до 10 × 10 24 10 −25 до 10 −23

Факторы, влияющие на электропроводность

На проводимость или удельное сопротивление материала влияют три основных фактора:

  1. Площадь поперечного сечения: Если поперечное сечение материала велико, через него может проходить больший ток.Точно так же тонкое поперечное сечение ограничивает ток.
  2. Длина проводника: Короткий проводник позволяет току течь с большей скоростью, чем длинный провод. Это немного похоже на попытку переместить множество людей через коридор.
  3. Температура: Повышение температуры заставляет частицы вибрировать или больше двигаться. Увеличение этого движения (повышение температуры) снижает проводимость, потому что молекулы с большей вероятностью будут мешать прохождению тока.При экстремально низких температурах некоторые материалы становятся сверхпроводниками.

Ресурсы и дополнительная информация

Таблица удельного сопротивления / Диаграмма для обычных материалов

Таблица удельного электрического сопротивления материалов, которые могут использоваться в электрических и электронных компонентах, включая удельное сопротивление меди, удельное сопротивление латуни и удельное сопротивление алюминия.


Resistance Tutorial:
Что такое сопротивление Закон Ома Омические и неомические проводники Сопротивление лампы накаливания Удельное сопротивление Таблица удельного сопротивления для распространенных материалов Температурный коэффициент сопротивления Электрическая проводимость Последовательные и параллельные резисторы Таблица параллельных резисторов


Приведенная ниже таблица удельного электрического сопротивления содержит значения удельного сопротивления для многих веществ, широко используемых в электронике.В частности, он включает удельное сопротивление меди, удельное сопротивление алюминия, золота и серебра.

Удельное электрическое сопротивление особенно важно, поскольку оно определяет его электрические характеристики и, следовательно, пригодность его для использования во многих электрических компонентах. Например, будет видно, что удельное сопротивление меди, удельное сопротивление алюминия и серебра и золота определяет, где используются эти металлы.

Чтобы сравнить способность различных материалов проводить электрический ток, используются значения удельного сопротивления.

Что означают цифры удельного сопротивления

Чтобы иметь возможность сравнивать удельное сопротивление различных материалов, таких как медь и серебро, и других металлов и веществ, включая висмут, латунь и даже полупроводники, необходимо использовать стандартное измерение.

Определение удельного сопротивления гласит, что удельное сопротивление вещества — это сопротивление куба этого вещества, имеющего края единичной длины, при том понимании, что ток течет перпендикулярно противоположным граням и равномерно распределяется по ним.

Удельное сопротивление обычно измеряется в Омметрах. Это означает, что удельное сопротивление измеряется для куба материала размером метр в каждом направлении.

Таблица удельного сопротивления для обычных материалов

В таблице ниже приведены значения удельного сопротивления для различных материалов, в частности металлов, используемых в качестве проводящих электричество.

Значения удельного сопротивления даны для материалов, включая медь, серебро, золото, алюминий, латунь и т.п.


Таблица удельного электрического сопротивления обычных материалов
Материал Удельное электрическое сопротивление при 20 ° C
Ом · м
Алюминий 2.8 х 10 -8
Сурьма 3,9 x 10 -7
висмут 1,3 x 10 -6
Латунь ~ 0,6 — 0,9 x 10 -7
кадмий 6 x 10 -8
Кобальт 5.6 х 10 -8
Медь 1,7 x 10 -8
Золото 2,4 x 10 -8
Углерод (графит) 1 x 10 -5
Германий 4,6 х 10 -1
Утюг 1.0 х 10 -7
Свинец 1,9 x 10 -7
Манганин 4,2 x 10 -7
нихром 1,1 x 10 -6
Никель 7 x 10 -8
Палладий 1,0 x 10 -7
Платина 0.98 х 10 -7
Кварц 7 х 10 17
Кремний 6,4 х 10 2
Серебро 1,6 x 10 -8
тантал 1,3 x 10 -7
Олово 1,1 x 10 -7
Вольфрам 4.9 х 10 -8
цинк 5,5 x 10 -8

Удельное сопротивление материалов — лучшее

Видно, что удельное сопротивление меди и удельное сопротивление латуни низкое, и ввиду их стоимости по сравнению с серебром и золотом они становятся экономически эффективными материалами для использования во многих проводах. Удельное сопротивление меди и простота ее использования означают, что она также используется почти исключительно в качестве проводящего материала на печатных платах.

Алюминий, в особенности медь, иногда используется из-за их низкого удельного сопротивления. Большая часть проводов, используемых в наши дни для межсоединений, сделана из меди, так как она обеспечивает низкий уровень удельного сопротивления при приемлемой стоимости.

Удельное сопротивление золота также важно, потому что золото используется в некоторых критических областях, несмотря на его стоимость. Часто позолота встречается на высококачественных слаботочных разъемах, где оно обеспечивает наименьшее контактное сопротивление. Золотое покрытие очень тонкое, но даже в этом случае оно способно обеспечить требуемые характеристики в разъемах.

Серебро

имеет очень низкий уровень удельного сопротивления, но оно не так широко используется из-за его стоимости и тусклости, что может привести к более высокому контактному сопротивлению. Оксид может действовать как выпрямитель при некоторых обстоятельствах, которые могут вызвать некоторые неприятные проблемы в радиочастотных цепях, генерируя так называемые пассивные продукты интермодуляции.

Однако он использовался в некоторых катушках для радиопередатчиков, где низкое электрическое сопротивление серебра уменьшало потери. При использовании в этом приложении он обычно наносился только на существующий медный провод — скин-эффект, влияющий на высокочастотные сигналы, означал, что только поверхность провода использовалась для проведения высокочастотных электрических токов.Покрытие проволоки серебром значительно снизило затраты по сравнению с сплошной серебряной проволокой без какого-либо значительного снижения производительности.

Другие материалы в таблице удельного электрического сопротивления могут не иметь такого очевидного применения. Тантал присутствует в таблице, потому что он используется в конденсаторах — никель и палладий используются в торцевых соединениях многих компонентов для поверхностного монтажа, таких как конденсаторы.

Кварц находит основное применение в качестве пьезоэлектрического резонансного элемента. Кристаллы кварца используются в качестве элементов определения частоты во многих генераторах, где его высокое значение Q позволяет создавать схемы с очень стабильной частотой.Они аналогичным образом используются в высокоэффективных фильтрах. Кварц имеет очень высокий уровень удельного сопротивления и не является хорошим проводником электричества, будучи классифицированным как изолятор.

Классификация удельного сопротивления проводников, изоляторов, полупроводников

Существует три широких классификации материалов с точки зрения их удельного сопротивления: проводники, полупроводники и изоляторы.


Сравнение удельного сопротивления проводников, полупроводников и изоляторов
Материал Типичный диапазон удельного сопротивления (Ом · м)
Проводники 10 -2 -10 -8
Полупроводники 10 -6 -10 6
Изоляторы 10 11 -10 19

Эти цифры являются ориентировочными.Показатели для полупроводников будут сильно зависеть от уровня легирования.

Удельное электрическое сопротивление материалов является ключевым электрическим параметром. Он определяет, можно ли эффективно использовать материалы во многих электрических и электронных приложениях. Это ключевой параметр, который используется для определения материалов, которые будут использоваться в электрических и электронных элементах.

Другие основные концепции электроники:
Напряжение Текущий Мощность Сопротивление Емкость Индуктивность Трансформеры Децибел, дБ Законы Кирхгофа Q, добротность Радиочастотный шум
Вернуться в меню «Основные понятия электроники».. .

Таблица удельного электрического сопротивления и проводимости

Резистор имеет высокое электрическое сопротивление, а проводник — высокую проводимость. (Николас Томас)

Это таблица удельного электрического сопротивления и электропроводности нескольких материалов. Включены металлы, элементы, вода и изоляторы.

Удельное электрическое сопротивление, обозначаемое греческой буквой ρ (ро), является мерой того, насколько сильно материал препятствует прохождению электрического тока.Чем ниже удельное сопротивление, тем легче материал пропускает электрический заряд. Чем выше удельное сопротивление, тем труднее течь току. Материалы с высоким удельным сопротивлением представляют собой электрические резисторы.

Электропроводность — величина, обратная удельному сопротивлению. Электропроводность — это мера того, насколько хорошо материал проводит электрический ток. Материалы с высокой электропроводностью являются электрическими проводниками. Электропроводность может быть представлена ​​греческой буквой σ (сигма), κ (каппа) или γ (гамма).

Таблица удельного сопротивления и проводимости при 20 ° C

Материал ρ (Ом • м) при 20 ° C
Удельное сопротивление
σ (См / м) при 20 ° C
Электропроводность
Серебро 1,59 × 10 −8 6,30 × 10 7
Медь 1,68 × 10 −8 5,96 × 10 7
Медь отожженная 1.72 × 10 −8 5,80 × 10 7
Золото 2,44 × 10 −8 4,10 × 10 7
Алюминий 2,82 × 10 −8 3,5 × 10 7
Кальций 3,36 × 10 −8 2,98 × 10 7
Вольфрам 5,60 × 10 −8 1,79 × 10 7
Цинк 5.90 × 10 −8 1,69 × 10 7
Никель 6,99 × 10 −8 1,43 × 10 7
Литий 9,28 × 10 −8 1,08 × 10 7
Железо 1,0 × 10 −7 1,00 × 10 7
Платина 1,06 × 10 −7 9,43 × 10 6
Олово 1.09 × 10 −7 9,17 × 10 6
Углеродистая сталь (10 10 ) 1,43 × 10 −7
Свинец 2,2 × 10 — 7 4,55 × 10 6
Титан 4,20 × 10 −7 2,38 × 10 6
Текстурированная электротехническая сталь 4,60 × 10 −7 2,17 × 10 6
Манганин 4.82 × 10 −7 2,07 × 10 6
Константан 4,9 × 10 −7 2,04 × 10 6
Нержавеющая сталь 6,9 × 10 — 7 1,45 × 10 6
Меркурий 9,8 × 10 −7 1,02 × 10 6
Нихром 1,10 × 10 −6 9,09 × 10 5
GaAs 5 × 10 −7 до 10 × 10 −3 5 × 10 −8 до 10 3
Углерод (аморфный) 5 × 10 −4 до 8 × 10 −4 1.От 25 до 2 × 10 3
Углерод (графит) 2,5 × 10 −6 до 5,0 × 10 −6 // базисная плоскость
3,0 × 10 −3 ⊥базальная плоскость
От 2 до 3 × 10 5 // базальная плоскость
3,3 × 10 2 ⊥базальная плоскость
Углерод (алмаз) 1 × 10 12 ~ 10 −13
Германий 4,6 × 10 −1 2,17
Морская вода 2 × 10 −1 4.8
Питьевая вода 2 × 10 1 до 2 × 10 3 5 × 10 −4 до 5 × 10 −2
Кремний 6,40 × 10 2 1,56 × 10 −3
Дерево (влажное) 1 × 10 3 до 4 10 −4 до 10 -3
Деионизированная вода 1,8 × 10 5 5,5 × 10 −6
Стекло 10 × 10 10 до 10 × 10 14 10 −11 до 10 −15
Твердая резина 1 × 10 13 10 −14
Древесина (сушка в печи) 1 × 10 14 до 16 10 −16 до 10 -14
Сера 1 × 10 15 10 −16 9 0040
Воздух 1.3 × 10 16 до 3,3 × 10 16 3 × 10 −15 до 8 × 10 −15
Парафиновый воск 1 × 10 17 10 −18
Плавленый кварц 7,5 × 10 17 1,3 × 10 −18
ПЭТ 10 × 10 20 10 −21
Тефлон 10 × 10 22 до 10 × 10 24 10 −25 до 10 −23

Факторы, влияющие на электрическую проводимость

Есть три основных фактора, которые влияют на проводимость или удельное сопротивление материала:

  1. Площадь поперечного сечения: Если поперечное сечение материала велико, он может позволить большему току проходить через него.Точно так же тонкое поперечное сечение ограничивает ток. Например, толстая проволока имеет большее поперечное сечение, чем тонкая проволока.
  2. Длина проводника: Короткий проводник позволяет току течь с большей скоростью, чем длинный провод. Это похоже на попытку провести через коридор множество людей по сравнению с дверью.
  3. Температура: Повышение температуры заставляет частицы вибрировать или больше двигаться. Увеличение этого движения (повышение температуры) снижает проводимость, потому что молекулы с большей вероятностью будут мешать прохождению тока.При экстремально низких температурах некоторые материалы становятся сверхпроводниками.

Список литературы

  • Гленн Элерт (ред.). «Удельное сопротивление стали». Сборник фактов по физике.
  • Данные о свойствах материалов MatWeb.
  • Оринг, Милтон (1995). Инженерные материалы е, Том 1 (3-е изд.). п. 561.
  • Pawar, S.D .; Муругавел, П .; Лал, Д. М. (2009). «Влияние относительной влажности и давления на уровне моря на электропроводность воздуха над Индийским океаном». Журнал геофизических исследований 114: D02205.

Удельное сопротивление (ρ) и проводимость (σ) металлов, сплавов, горных пород и грунтов

Контент левой границы — RF Cafe

Авторские права: 1996-2024
Web master:
Юбка Blatten berger ,

BSEE — KB3UON

RF Cafe начало свою деятельность в 1996 году как «RF Tools» в веб-пространстве с псевдонимом AOL. 2 МБ.Его основная цель заключалась в том, чтобы предоставить мне быстрый доступ к обычно необходимым формулы и справочные материалы при выполнении моей работы в качестве ВЧ системы и схемы инженер-проектировщик. Всемирная паутина (Интернет) была в значительной степени неизвестной сущностью в время и пропускная способность были дефицитом. Модемы удаленного доступа стремительно развиваются со скоростью 14,4 кбит / с набирая телефонную линию, и приятный женский голос объявил: «У вас есть Почта »при поступлении нового сообщения …

Все товарные знаки, авторские права, патенты и другие права собственности на изображения и текст, используемый на сайте RF Cafe, настоящим подтверждением обрезан.

Сайт моего хобби:

Самолеты и Ракеты .com

Подзаголовок — RF Cafe

Удельное сопротивление, также именуемое как удельное сопротивление, зависит от природы материала, а также его объема определение (форма и размер).Удельное сопротивление выражается в единицах, которые являются произведением сопротивление и длина; например, Ом · см. Символ, наиболее часто используемый для обозначения удельного сопротивления. есть rho (ρ).

Электропроводность — это величина, обратная сопротивлению. Электропроводность выражается в единицах, являющихся частным от проводимость (Сименс) и длина; например, См / см. Символ, наиболее часто используемый для удельное сопротивление — сигма (σ).

В качестве примера расчета сопротивления объема рассмотрим рисунок слева.Предположим, что медный провод 12 AWG с удельным сопротивлением (из таблицы) 1,72×10 -6 Ом · см, площадь поперечного сечения (A) 0,03309 см 2 и длина 1 метр. По данной формуле его сопротивление составляет:

,

, что хорошо согласуется с типичными указанными значениями Ом / км, опубликованными производителями проводов. Alpha утверждает, что 1,59 Ом / 1000 ‘или 5,22 Ом / км.

Таблица Приведенные ниже значения удельного сопротивления взяты из справочных данных для радио. Инженеры , 1995, Самс Паблишинг.Пожалуйста, проверьте точность у другого источника. Видеть Таблица пород и грунтов внизу. Интересное примечание: никель-серебро соединение фактически не содержит серебра; его название происходит от серебристого цвета.

Алюмель цельный 33,3 0 0,0012
Алюминий жидкий
твердый
20,3
2,62
670
20

.0039
Сурьма жидкий
твердый
123
39,2
800
20

0,0036
мышьяк цельный 35 0 0,0042
Бериллий цельный 4,57 20
висмут жидкий
твердый
128.9
115
300
20

0,004
Бор цельный 1,8х10 12 0
Латунь (66 Cu, 34 Zn) цельный 3,9 20 0,002
кадмий жидкий
твердый
34
7,5
400
20

0.0038
Углерод алмаз
графит
графен
5х10 20
1400
15
20

-0,0005
Церий цельный 78 20
Цезий жидкий
твердый
36,6
20
30
20
Chromax (15 Cr, 35 Ni, остальное Fe) цельный 100 20 0.00031
хромель цельный 70-110 0 0,00011-0,000054
Хром цельный 2,6 0
Кобальт цельный 9,7 20 0,0033
константан (55 Cu, 45 Ni) цельный 44.2 20 0,0002
Медь (отожженная) жидкий
твердый
21,3
1,7241
1083
20

0,0039
Галлий твердый
жидкий
27
53
30
0
Золото жидкий
твердый
30.8
2,44
1063
20

0,0034
Гранит цельный 1×10 13 — 1×10 15
Гафний цельный 32,1 20
Индий жидкий
твердый
29
9
157
20

0.00498
Иридий цельный 5,3 20 0,0039
Утюг цельный 9,71 20 0,0052-0,0062
Ковар A (29, Ni, 17 Co, 0,3 Mn, остальное Fe) цельный 45-84 20
Поток лавы (основной)
Лава, свежая
жидкость 1×10 12 — 1×10 13
Свинец жидкий
твердый
98
21.9
400
20

0,004
ПБО 2 цельный 92
Литий жидкий
твердый
45
9,3
230
20
0,003
0,005
Магний цельный 4,46 20 0.004
Марганец цельный 5 20
MnO 2 цельный 6000000 20
Магнанин (84 Cu, 12 Mn, 4 Ni) цельный 44 20 ± 0,0002
Меркурий жидкий
твердый
95.8
21,3
20
-50
0,00089
молибден цельный 5,17
4,77
0
20

0,0033
Металлический монель (67 Ni, 30 Cu, 1,4 Fe) цельный 42 20 0,002
Неодим цельный 79 18
Нихром (65 Ni, 12 Cr, 23 Fe) цельный 100 20 0.00017
Никель цельный 6,9 20 0,0047
Никель-серебро (64 Cu, 18 Zn, 18 Ni) цельный 28 20 0,00026
Ниобий цельный 12,4 20
Осмий цельный 9 20 0.0042
Палладий цельный 10,8 20 0,0033
Фосфорная бронза (4 Sn, 0,5 P, остальное Cu) цельный 9,4 20 0,003
Платина цельный 10,5 20 0,003
Плутоний цельный 150 20
Калий жидкий
твердый
13
7
62
20

0.006
празеодим цельный 68 25
Рений цельный 19,8 20
Родий цельный 5,1 20 0,0046
Рубидий цельный 12,5 20
рутений цельный 10 20
Селен цельный 1.2 20
Серебро цельный 1,62 20 0,0038
Натрий жидкий
твердый
9,7
4,6
100
20
Сталь (0,4-0,5 C, остальное Fe) цельный 13–22 20 0.003
Сталь, марганец (13 Mn, 1 C, 86 Fe) цельный 70 20 0,001
Сталь, нержавеющая (0,1 C, 18 Cr, 8 Ni, остальное Fe) цельный 90 20
Стронций цельный 23 20
сера цельный 2х10 23 20
тантал цельный 13.1 20 0,003
Таллий цельный 18,1 20 0,004
торий цельный 18 20 0,0021
Олово цельный 11,4 20 0,0042
Титан цельный 47.8 25
Тофет A (80 Ni, 20 Cr) цельный 108 20 0,00014
Вольфрам цельный 5,48 20 0,0045
Вт 2 О 5 цельный 450 20
WO 3 цельный 2х10 11 20
Уран цельный 29 0 0.0021
цинк жидкий
твердый
35,3
6
420
20

0,0037
цирконий цельный 40 20 0,0044

Гранит 10 7 -10 9
Поток лавы (основной)
Лава, свежая
10 6 — 10 7
3×10 5 — 10 6
Мрамор
Мрамор, белый
Мрамор, желтый
4×10 8
10 10
10 10
Кварц, жила, массив> 10 6
Сланец, слюда 10 7
Сланец, пласт
Сланец, не такой
10 5
10 4
Известняк
Известняк, кембрийский
10 4
10 4 -10 5
Песчаник
Песчаник, восточный
10 5
3×10 3 -10 4
Глина синяя
Глина огненная
2х10 4
2х10 5
Глинистая земля 10 4 — 4х10 4
Гравий 10 5
Песок сухой
Песок влажный
10 5 — 10 6
10 6 — 10 5

Опубликовано: 13 июля, 2018

Нижний колонтитул — RF Cafe
Контент правой границы — RF Cafe

SMART и токопроводящие ткани, пряжа или ткани

Следующий JEC world пройдет с 8 по 10 марта 2022 года! … Сделайте перчатки тактильными! используйте нашу кондукторную швейную нить SILVERPAM

Металлические нагревательные или токопроводящие нити и гибкие конструкции для технического текстиля или композитов функционализация:

Мы проектируем и производим гибкую, металлическую, токопроводящую или нагревательную пряжу для передачи энергии или функциональности материалов.
Вы можете разместить их в тканях или встроить в гибкие конструкции или композиты.

Что мы подразумеваем под

Передача энергии :

  • Электроэнергия
  • Оптическая энергия
  • Тепловая энергия (передача, контролируемая материалами или жидкостями)

Что мы подразумеваем под

проводящими или резистивными волокнами :

  • ультратонкие волокна или комплексные волокна из сплавов металлов или нержавеющей стали;
  • Волокна металлические, привитые или с покрытием
  • Многокомпонентная пряжа с добавками термопластов или смол
  • Оптоволокно
  • Капилляры или микротрубки для теплоносителей

Что мы подразумеваем под

гибкими конструкциями :

  • Металл или нержавеющая сталь Устойчивые к высоким температурам микроволокна, ленты или пряжа:
  • В виде токопроводящих жил:
  • На основе гибких функциональных тканей:

Металлические нагревательные или проводящие волокна, пряжа и гибкие конструкции


для функционализации тканей или композитов SMART

Ориентация на несколько проводящих сырьевых материалов

Мы работаем с рядом ультратонких металлических или проводящих волокон, выбранных с учетом их особых свойств.

Трансверсальность: мы используем много технологий преобразования текстиля

Благодаря собственным производственным мощностям или известным партнерам мы оптимизируем свойства наших функциональных материалов для удовлетворения потребностей наших клиентов.

Работаем на трех основных рынках

    Нагревательные нити или ткани для функциональности многослойных или композитных деталей

    Гибкие элементы для электроники: смарт-текстиль, антенны RFID, связанная одежда, подключение

    Высокотемпературная фильтрация и катализ горячих газов

Во что мы верим:

«Самый большой инновационный потенциал лежит на перекрестке материалов, технологий и человека»

«Прошлые или будущие инновации очень часто вдохновляются тем, что уже существует в Природе!»

Электропроводность металлов, отсортированных по удельному сопротивлению

 Исходный код:
 1 - CSNDT
 2 - Руководство по вихретоковым испытаниям методом вихретокового контроля
 3 - Журнал NDT, сентябрь / октябрь 1955 г., статья Косгроува

СОПРОТИВЛЯТЬСЯ.КОНД. ИСТОЧНИК
Ом-м SIEMENS / м% КОД IACS МАТЕРИАЛ
-------------------------------------------------- --------------------------------
1.591E-08 6.287E + 07 108.40 1 Серебро, чистое
1.642E-08 6.090E + 07 105.00 2 Серебро, чистое
1.664E-08 6.009E + 07103.60 1 Медь, чистая
1.707E-08 5.858E + 07 101.00 1 Медь, электролитический вязкий пек (отожженный)
1.724E-08 5.800E + 07 100.00 2 Медь, чистая
2.028E-08 4.930E + 07 85.00 1 Медь раскисленная (отожженная)
2.349E-08 4.257E + 07 73.40 1 Золото
2.463E-08 4.060E + 07 70.00 2 Золото, чистое
2.655E-08 3.767E + 07 64.94 1 Алюминий, 99,99%
2.826E-08 3.538E + 07 61.00 2 Алюминий чистый
2.871E-08 3.483E + 07 60.00 - 60.10 3 Алюминиевый сплав, 7072
2.903E-08 3.445E + 07 57.00 - 61.80 3 Алюминиевый сплав, 1100
2.922E-08 3.422E + 07 59.00 1 Алюминий, 2S Конд. «0»
3.025E-08 3.306E + 07 57.00 1 Алюминий, 2S Конд. h28
3.073E-08 3.254E + 07 55.70 - 56.50 3 Алюминиевый сплав, 6951-0
3.079E-08 3.248E + 07 56.00 1 Позолоченный металл (отожженный)
3.135E-08 3.190E + 07 55.00 1 Алюминий, A51S Cond. «0»
3.184E-08 3.141E + 07 53.30 - 55.00 3 Алюминиевый сплав, 6151-0
3.235E-08 3.091E + 07 52.30 - 54.30 3 Алюминиевый сплав, 4043-F
3.250E-08 3.077E + 07 53.00 - 53.10 3 Алюминиевый сплав, 6951-F
3.281E-08 3.048E + 07 52.30 - 52.80 3 Алюминиевый сплав, 5005
3.435E-08 2.912E + 07 50.10 - 50.30 3 Алюминиевый сплав, X3005-0
3.448E-08 2.900E + 07 50.00 1 Алюминий, 24S Конд. «0»
3.448E-08 2.900E + 07 50.00 1 Алюминий, 3S Cond. «0»
3.448E-08 2.900E + 07 50.00 1 Алюминий, 18S Конд. «0»
3.448E-08 2.900E + 07 50.00 1 Алюминий, 14S Конд. «0»
3.473E-08 2.880E + 07 48,60 - 50,70 3 Алюминиевый сплав, 2014-F и -0
3.490E-08 2.865E + 07 49.30 - 49.50 3 Алюминиевый сплав, 2017-F
3.515E-08 2.845E + 07 48.30 - 49.80 3 Алюминиевый сплав, 5050
3.519E-08 2.842E + 07 47.00 - 51.00 3 Алюминиевый сплав, 6062-F
3.540E-08 2.825E + 07 48.70 1 Кальций
3.592E-08 2.784E + 07 48.00 1 Bronze Phos., 1,25% Phos. Оценка E
3.592E-08 2.784E + 07 48.00 1 Phos.Бронза, 1,25% Phos. Оценка E
3.618E-08 2.764E + 07 46.80 - 48.50 3 Алюминиевый сплав, 2024-F
3.649E-08 2.741E + 07 44.70 - 49.80 3 Алюминиевый сплав, 3003-0
3.661E-08 2.732E + 07 44.70 - 49.50 3 Алюминиевый сплав, 6062-T6
3.736E-08 2.677E + 07 44.50 - 47.80 3 Алюминиевый сплав, 7075-F
3.769E-08 2.654E + 07 45.50 - 46.00 3 Алюминиевый сплав, X7178-F и -0
3.798E-08 2.633E + 07 42.30 - 48.50 3 Алюминиевый сплав, 6061-F и -0
3.831E-08 2.610E + 07 45.00 1 Алюминий, 17S Конд. «0»
3.831E-08 2.610E + 07 45.00 1 Алюминий, 53S Конд. «0»
3.831E-08 2.610E + 07 45.00 1 Алюминий, 61S Конд. «0»
3.831E-08 2.610E + 07 45.00 1 Алюминий, A51S Cond. Т4 и Т6
3.831E-08 2.610E + 07 45.00 1 Алюминиевый сплав, 750
3.861E-08 2.590E + 07 42.30 - 47.00 3 Алюминиевый сплав, 5357
3.861E-08 2.590E + 07 37.80 - 51.50 3 Алюминиевый сплав, 3003-h24 и -h22
3.879E-08 2.578E + 07 43.90 - 45.00 3 Алюминиевый сплав, 6151-T6
3.918E-08 2.552E + 07 44.00 1 бронза, коммерческая (отожженная)
3.918E-08 2.552E + 07 44.00 1 Алюминиевый сплав, 142 Sand Cond. T21
3.941E-08 2.538E + 07 43.50 - 44.00 3 Алюминиевый сплав, 6062-T4
3.950E-08 2.532E + 07 39.30 - 48.00 3 Алюминиевый сплав, 6053
4.000E-08 2.500E + 07 43.10 1 Бериллий
4.010E-08 2.494E + 07 43.00 1 Алюминиевый сплав, 355 Sand Cond.T51
4.010E-08 2.494E + 07 43.00 1 Алюминиевый сплав, 356 Sand Cond. T51
4.043E-08 2.474E + 07 37.80 - 47.50 3 Алюминиевый сплав, 3003-h34 и -h38
4.066E-08 2.459E + 07 40.00 - 44.80 3 Алюминиевый сплав, 6061-T6 и -T9
4.066E-08 2.459E + 07 41.50 - 43.30 3 Алюминиевый сплав, 6151-T4
4.081E-08 2.451E + 07 42.10 - 42.40 3 Алюминиевый сплав, 2127-T4
4.105E-08 2.436E + 07 42.00 1 Алюминиевый сплав, 355 Sand Cond.T7
4.105E-08 2.436E + 07 42.00 1 Алюминиевый сплав, 43 (отожженный)
4.105E-08 2.436E + 07 42.00 1 Алюминий, 3S Cond. H 12
4.105E-08 2.436E + 07 42.00 1 Бронза, коммерческий свинец
4.105E-08 2.436E + 07 42.00 1 Коммерческая бронза с содержанием свинца
4.160E-08 2.404E + 07 39.40 - 43.50 3 Алюминиевый сплав, 3004
4.205E-08 2.378E + 07 41.00 1 Алюминий, 3S Cond. H 14
4.205E-08 2.378E + 07 41.00 1 Алюминиевый сплав, 122 Sand Cond.Т2
4.289E-08 2.332E + 07 40.20 3 Алюминиевый сплав, 2618
4.310E-08 2.320E + 07 40.00 1 Алюминий, 24S Конд. T6
4.310E-08 2.320E + 07 40.00 1 Алюминий, 18S Конд. T61
4.310E-08 2.320E + 07 40.00 1 Алюминий, 11S Конд. Т3
4.310E-08 2.320E + 07 40.00 1 Алюминий, 14S Конд. T6
4.310E-08 2.320E + 07 40.00 1 Алюминий, 3S Cond. H 18
4.310E-08 2.320E + 07 40.00 1 Алюминий, 32S Конд. «0»
4.310E-08 2.320E + 07 40.00 1 Алюминий, 53S Конд. Т4 и Т6
4.310E-08 2.320E + 07 40.00 1 Алюминий, 61S Cond. Т4 и Т6
4.415E-08 2.265E + 07 37.60 - 40.50 3 Алюминиевый сплав, 6061-T4
4.421E-08 2.262E + 07 39.00 1 Алюминиевый сплав, 356 Sand Cond. T6
4.421E-08 2.262E + 07 39.00 1 Алюминиевый сплав, 355 Пермь. Mold Cond. T6
4.421E-08 2.262E + 07 39.00 1 Алюминиевый сплав, 13
4.432E-08 2.256E + 07 38.90 1 Бериллий
4.438E-08 2.253E + 07 38.00 - 39.70 3 Алюминиевый сплав, 2014-T6
4.467E-08 2.239E + 07 38.60 1 Магний, чистый
4.490E-08 2.227E + 07 38.40 1 Родий
4.610E-08 2.169E + 07 37.40 3 Алюминиевый сплав, 2218-T61
4.660E-08 2.146E + 07 37.00 1 Алюминиевый сплав, 142 Sand Cond. T77
4.660E-08 2.146E + 07 37.00 1 Алюминиевый сплав, 195 конд. T62
4.660E-08 2.146E + 07 37.00 2 Магний
4.660E-08 2.146E + 07 37.00 1 Алюминиевый сплав, 360
4.660E-08 2.146E + 07 37.00 1 Алюминиевый сплав, 355 Sand Cond. T61
4.660E-08 2.146E + 07 37.00 1 Алюминиевый сплав, 43 литой
4.660E-08 2.146E + 07 37.00 1 Алюминиевый сплав, A 108
4.660E-08 2.146E + 07 37.00 1 Латунь, красная (отожженная)
4.756E-08 2.103E + 07 36.00 - 36.50 3 Алюминиевый сплав, 2011-T3
4.789E-08 2.088E + 07 36.00 1 Алюминиевый сплав, B 195 Cond.T6
4.789E-08 2.088E + 07 36.00 1 Цельнолитой алюминий, конд. Sol. H.T. И стресс
4.789E-08 2.088E + 07 36.00 1 Алюминиевый сплав, 355 Sand Cond. T6
4.816E-08 2.076E + 07 35.30 - 36.30 3 Алюминиевый сплав, 4032-T6
4.843E-08 2.065E + 07 33.60 - 37.60 3 Алюминиевый сплав, 5052
4.926E-08 2.030E + 07 35.00 1 Алюминиевый сплав, 195 конд. Т4
4.926E-08 2.030E + 07 35.00 1 Алюминиевый сплав, 214
4.926E-08 2.030E + 07 35.00 1 Алюминиевый сплав, 40E
4.926E-08 2.030E + 07 35.00 1 Алюминий, 52S Конд. «0» и H 38
4.926E-08 2.030E + 07 35.00 1 Алюминий, 32S Конд. T6
4.926E-08 2.030E + 07 35.00 1 Алюминиевый сплав, B 195 Cond. Т4
4.998E-08 2.001E + 07 34.50 1 Магний (деформируемые сплавы)
5.071E-08 1.972E + 07 34.00 1 Алюминиевый сплав, 142 Sand Cond. T571
5.071E-08 1.972E + 07 34.00 1 Алюминиевый сплав, 122 Пермь.Форма как литая
5.124E-08 1.952E + 07 32.50 - 34.80 3 Алюминиевый сплав, 2014-T3 и -T4
5.209E-08 1.920E + 07 31.40 - 34.80 3 Алюминиевый сплав, 7075-T6
5.225E-08 1.914E + 07 33.00 1 Молибден
5.225E-08 1.914E + 07 33.00 1 Алюминиевый сплав, 122 Sand Cond. T61
5.225E-08 1.914E + 07 33.00 1 Алюминиевый сплав, A214
5.289E-08 1.891E + 07 32.60 1 Иридий
5.330E-08 1.876E + 07 28.60 - 36.10 3 Алюминиевый сплав, 2024-T3
5.388E-08 1.856E + 07 32.00 1 Латунь, низкая (отожженная)
5.388E-08 1.856E + 07 32.00 1 Алюминиевый сплав, 142 Пермь. Mold Cond. T61
5.388E-08 1.856E + 07 27.00 - 37.00 3 Алюминиевый сплав, 7075-W
5.388E-08 1.856E + 07 32.00 2 Алюминиевый сплав, 7075-T6
5.448E-08 1.836E + 07 30.50 - 32.80 3 Алюминиевый сплав, 5154
5.491E-08 1.821E + 07 31.40 1 Вольфрам
5.562E-08 1.798E + 07 31.00 1 Алюминиевый сплав, 108
5.747E-08 1.740E + 07 30.00 1 Алюминий, 24S Конд. Т4
5.747E-08 1.740E + 07 30.00 1 Алюминий Allcast, Sol H.T. и в возрасте
5.747E-08 1.740E + 07 30.00 1 Алюминий, 17S Конд. Т4
5.747E-08 1.740E + 07 30.00 1 Алюминиевый сплав, 113
5.747E-08 1.740E + 07 30.00 1 Алюминиевый сплав, R 317
5.747E-08 1.740E + 07 30.00 1 Алюминий, 75S Конд. T6
5.747E-08 1.740E + 07 30.00 1 Цельнолитой алюминий со снятым напряжением
5.766E-08 1.734E + 07 28.80 - 31.00 3 Алюминиевый сплав, 2024-T4
5.805E-08 1.723E + 07 26.80 - 32.60 3 Алюминиевый сплав, X7178-W и T6
5.884E-08 1.699E + 07 29.10 - 29.50 3 Алюминиевый сплав, 2024-T36
5.945E-08 1.682E + 07 29.00 1 Алюминиевый сплав, A 132 Cond. T551
5.945E-08 1.682E + 07 29.00 1 Алюминий, красный X-8 Cond. Снятие стресса
5.945E-08 1.682E + 07 29.00 1 Алюминий, 56S Конд. «0»
5.945E-08 1.682E + 07 29.00 2 Цинк
5.956E-08 1.679E + 07 28.10 - 29.80 3 Алюминиевый сплав, 5056
6.158E-08 1.624E + 07 28.00 1 Цинк, товарный прокат
6.158E-08 1.624E + 07 28.00 1 Алюминиевый сплав, 319 Пермь. Плесень
6.158E-08 1.624E + 07 28.00 1 Патрон латунный (отожженный)
6.158E-08 1.624E + 07 28.00 1 Muntz Metal (отожженный)
6.158E-08 1.624E + 07 28.00 1 Алюминиевый сплав, 85
6.158E-08 1.624E + 07 28.00 1 Латунь, картридж (отожженный)
6.247E-08 1.601E + 07 27.60 1 Кобальт
6.386E-08 1.566E + 07 27.00 1 Алюминиевый сплав, C113
6.386E-08 1.566E + 07 27.00 1 Цинк, литье под давлением
6.386E-08 1.566E + 07 27.00 1 Алюминий, 56S Конд. H 38
6.386E-08 1.566E + 07 27.00 1 Алюминий Allcast, литой
6.386E-08 1.566E + 07 27.00 1 Алюминиевый сплав, 319 Песок
6.386E-08 1.566E + 07 27.00 1 Латунь, желтая (отожженная)
6.386E-08 1.566E + 07 27.00 1 алюминиевый сплав, 380
6.631E-08 1.508E + 07 26.00 1 Латунь, с низким содержанием свинца (отожженная)
6.631E-08 1.508E + 07 26.00 1 Латунь, свинцовая морская (отожженная)
6.631E-08 1.508E + 07 26.00 1 Латунь, военно-морская (отожженная)
6.631E-08 1.508E + 07 26.00 1 Алюминиевый сплав, красный X-8 в литом виде
6.842E-08 1.462E + 07 25.20 1 Кадмий
6.842E-08 1.462E + 07 25.20 1 Никель, чистый (электролитический)
6.897E-08 1.450E + 07 25.00 1 свинцовая желтая латунь
6.897E-08 1.450E + 07 25.00 1 Цинк, литье под давлением
6.897E-08 1.450E + 07 25.00 1 Латунь, желтый свинец
7.009E-08 1.427E + 07 24.60 1 Admiralty Metal (отожженный)
7.184E-08 1.392E + 07 24.00 2 Латунь, Адмиралтейство
7.184E-08 1.392E + 07 24.00 1 Алюминиевый сплав, 218
7.184E-08 1.392E + 07 24.00 1 Марганцевая бронза (отожженная)
7.184E-08 1.392E + 07 24.00 2 Адмиралтейство Латунь
7.184E-08 1.392E + 07 24.00 1 Марганцевая бронза (отожженная)
7.496E-08 1.334E + 07 23.00 1 Латунь, алюминий (отожженный)
7.496E-08 1.334E + 07 23.00 1 Алюминиевая латунь (отожженная)
7.595E-08 1.317E + 07 22.70 1 Рутений
8.210E-08 1.218E + 07 21.00 1 Алюминиевый сплав, 220
8.210E-08 1.218E + 07 21.00 1 Бериллиевая медь, конд. В
8.210E-08 1.218E + 07 21.00 1 Медь бериллий, конд. В
8.535E-08 1.172E + 07 20.20 1 Литий
9.473E-08 1.056E + 07 18.20 1 Осмий
9.579E-08 1.044E + 07 18.00 1 Никель "А"
9.579E-08 1.044E + 07 18.00 1 Phos. Бронза, 5% Phos. Оценка отлично
9.579E-08 1.044E + 07 18.00 2 Утюг
9.579E-08 1.044E + 07 18.00 1 Латунь, полукрасный свинец
9.579E-08 1.044E + 07 18.00 1 Свинцовый полукрасный латунь
9.579E-08 1.044E + 07 18.00 1 Bronze Phos., 5% Phos.Оценка отлично
9.852E-08 1.015E + 07 17.50 1 бронзовый алюминий, 5% алюминия (отожженный)
9.852E-08 1.015E + 07 17.50 1 Алюминий - бронза, 5% алюминия (отожженный)
1.002E-07 9.976E + 06 17.20 1 Магний, A231
1.014E-07 9.860E + 06 17.00 1 Бериллиевая медь, конд. А
1.014E-07 9.860E + 06 17.00 1 Медь бериллий, конд. "А"
1.039E-07 9.628E + 06 16.60 1 Серебро, оловянный припой
1.039E-07 9.628E + 06 16.60 1 олово, серебряный припой
1.039E-07 9.628E + 06 16.60 1 Припой, олово серебро
1.059E-07 9.442E + 06 16.28 1 Платина
1.078E-07 9.280E + 06 16.00 1 Палладий
1.105E-07 9.048E + 06 15.60 1 Игнот железа (99,9% Fe)
1.105E-07 9.048E + 06 15.60 1 Слиток железа (99,9% Fe)
1.149E-07 8.700E + 06 15.00 1 Олово, чистое
1.149E-07 8.700E + 06 15.00 1 Магниевые сплавы (литые)
1.181E-07 8.468E + 06 14.60 1 Магний, A2 80
1.197E-07 8.352E + 06 14.40 1 Селен
1.232E-07 8.120E + 06 14.00 1 Бронза, свинцовое олово
1.232E-07 8.120E + 06 14.00 1 Оловянная бронза с содержанием свинца
1.232E-07 8.120E + 06 14.00 1 Олово (свинец), бронза
1.232E-07 8.120E + 06 14.00 1 Алюминий - бронза
1.232E-07 8.120E + 06 14.00 1 Бронза Алюминий
1.240E-07 8.062E + 06 13.90 1 Тантал
1.268E-07 7.888E + 06 13.60 1 Никель-платиновые сплавы
1.268E-07 7.888E + 06 13.60 1 Платина - никелевые сплавы
1.306E-07 7.656E + 06 13.20 1 Columbium
1.326E-07 7.540E + 06 13.00 1 Phos. Бронза, 8% Phos. Оценка C
1.326E-07 7.540E + 06 13.00 1 Bronze Phos., 8% Phos. Оценка C
1.347E-07 7.424E + 06 12.80 1 Магний, A251
1.368E-07 7.308E + 06 12.60 1 Алюминий - бронза, 10% алюминия (отожженный)
1.368E-07 7.308E + 06 12.60 1 бронзовый алюминий, 10% алюминия (отожженный)
1.379E-07 7.250E + 06 12.50 1 Магний, T454
1.402E-07 7.134E + 06 12.30 1 Магний, A261
1.437E-07 6.960E + 06 12.00 1 Бронза, кремний типа B (отожженный)
1.437E-07 6.960E + 06 12.00 1 Кремниевая бронза, тип B (отожженная)
1.437E-07 6.960E + 06 12.00 1 Латунь, высокопрочная желтая
1.449E-07 6.902E + 06 11.90 1 оловянный припой с сурьмой
1.449E-07 6.902E + 06 11.90 1 оловянный припой (сурьма)
1.449E-07 6.902E + 06 11.90 1 Припой, сурьмяное олово
1.486E-07 6.728E + 06 11.60 1 Платина, коммерческая
1.553E-07 6.438E + 06 11.10 1 Белый металл
1.567E-07 6.380E + 06 11.00 1 Phos. Бронза, 10% Phos. Оценка D
1.567E-07 6.380E + 06 11.00 1 Подшипник из олова из бронзы и свинца
1.567E-07 6.380E + 06 11.00 2 Бронза, Phos.
1.567E-07 6.380E + 06 11.00 1 Bronze Phos., 10% Phos. Оценка D
1.567E-07 6.380E + 06 11.00 2 Phos. Бронза
1.567E-07 6.380E + 06 11.00 1 Оловянный подшипник с содержанием свинца, бронза
1.567E-07 6.380E + 06 11.00 1 припой, 50-50 мягкий
1.596E-07 6.264E + 06 10.80 1 Магний, AZ80BTA
1.611E-07 6.206E + 06 10.70 1 Сталь, литая
1.759E-07 5.684E + 06 9.80 1 припой, 20-80 мягкий
1.771E-07 5.647E + 06 9.74 4 Медь 90%, никель 10%
1.895E-07 5.278E + 06 9.10 1 Платино-иридиевые сплавы
1.895E-07 5.278E + 06 9.10 1 Иридий-платиновые сплавы
1.916E-07 5.220E + 06 9.00 1 Магниевые литейные сплавы
1.959E-07 5.104E + 06 8.80 1 припой, 5-95 мягкий
1.959E-07 5.104E + 06 8.80 1 Хром
2.053E-07 4.872E + 06 8.40 2 свинца
2.077E-07 4.814E + 06 8.30 1 Свинец, Корродин
2.077E-07 4.814E + 06 8.30 1 Корродин Свинец
2.188E-07 4.570E + 06 7.88 1 Свинец, 1% сурьмы (закаленная и выдержанная)
2.188E-07 4.570E + 06 7.88 1 Сурьма Свинец, 1% (закаленная и выдержанная)
2.239E-07 4.466E + 06 7.70 1 Свинец, твердый (закаленный и выдержанный)
2.330E-07 4.292E + 06 7.40 1 Никель-платиновые сплавы
2.330E-07 4.292E + 06 7.40 1 Платина - никелевые сплавы
2.463E-07 4.060E + 06 7.00 1 Кремниевая бронза, тип A (отожженная)
2.463E-07 4.060E + 06 7.00 1 Бронза, кремний типа A (отожженный)
2,612E-07 3,828E + 06 6,60 1 Ванадий
2.874E-07 3.480E + 06 6.00 1 Серебро, 18% никель, сплав A
2.874E-07 3.480E + 06 6.00 1 Уран
2.874E-07 3.480E + 06 6.00 1 Никель, 18% никель Sil
2.874E-07 3.480E + 06 6.00 1 Баббит, свинцовая база
3.135E-07 3.190E + 06 5.50 1 Платина - рутений (ювелирный сорт)
3.135E-07 3.190E + 06 5.50 1 Рутений - Платина (ювелирный сорт)
3.316E-07 3.016E + 06 5.20 1 Сплавы платины и иридия, 18% никеля
3.316E-07 3.016E + 06 5.20 1 Иридий-платиновые сплавы, 18% никелевого серебра
3,748E-07 2,668E + 06 4,60 1 Никель 30% - Купро
3,748E-07 2,668E + 06 4,60 1 Купро-никель 30%
3.831E-07 2.610E + 06 4.50 2 Никель 30%, медь 70%
3.831E-07 2.610E + 06 4.50 2 Медь 70%, никель 30%
3.918E-07 2.552E + 06 4.40 1 Сурьма
4.105E-07 2.436E + 06 4.20 1 Олово, фольга
4.105E-07 2.436E + 06 4.20 1 Цирконий
4.310E-07 2.320E + 06 4.00 1 Рутений-платина (контактная степень)
4.310E-07 2.320E + 06 4.00 1 Платина - рутений (контактный сорт)
4.789E-07 2.088E + 06 3.60 2 Монель
4.816E-07 2.076E + 06 3.58 1 Монель
4.898E-07 2.042E + 06 3.52 1 Константан
5.071E-07 1.972E + 06 3.40 2 Цирконий
5.562E-07 1.798E + 06 3.10 2 Титан
5.945E-07 1.682E + 06 2.90 1 Сталь высоколегированная
6.897E-07 1.450E + 06 2.50 1 Сталь, нержавеющая сталь 304
6.897E-07 1.450E + 06 2.50 2 Сталь, нержавеющая сталь 304
7.184E-07 1.392E + 06 2.40 1 Сталь, 347 Нержавеющая сталь
7.184E-07 1.392E + 06 2.40 2 Циркалой - 2
7.496E-07 1.334E + 06 2.30 1 Сталь, нержавеющая сталь 316
7.837E-07 1.276E + 06 2.20 1 Титан
9.579E-07 1.044E + 06 1.80 1 Меркурий
9.796E-07 1.021E + 06 1.76 1 Инконель
1.014E-06 9.860E + 05 1.70 2 Инконель 600
1.149E-06 8.700E + 05 1.50 1 Хастеллой "D"
1.149E-06 8.700E + 05 1.50 2 Хастеллой "X"
1.232E-06 8.120E + 05 1.40 2 Васпалой
1.232E-06 8.120E + 05 1.40 1 Хастеллой "A"
1.326E-06 7.540E + 05 1.30 1 Хастеллой "B" и "C"
1.724E-06 5.800E + 05 1.00 2 Титан, 6АЛ-4В
7.837E-06 1.276E + 05 0.22 1 Графит 

6.8A: Электропроводность и удельное сопротивление

Удельное электрическое сопротивление и проводимость являются важными характеристиками материалов. У разных материалов разная проводимость и удельное сопротивление. Электропроводность основана на свойствах электрического переноса. Их можно измерить несколькими методами, используя различные инструменты. Если электричество легко проходит через материал, этот материал имеет высокую проводимость.Некоторые материалы с высокой проводимостью включают медь и алюминий. Электропроводность — это мера того, насколько легко электричество проходит через материал.

Зависимость проводимости от удельного сопротивления

Электропроводность и удельное сопротивление обратно пропорциональны друг другу. Когда проводимость низкая, сопротивление высокое. Когда удельное сопротивление низкое, проводимость высокая. Уравнение выглядит следующим образом:

\ [\ rho = \ dfrac {1} {\ sigma} \]

где

  • Удельное сопротивление обозначается как \ (\ rho \) и измеряется в Ом-метрах (\ (Ом · м \)),
  • Электропроводность обозначается как \ (\ sigma \) и измеряется в Siemens (\ (1 / Ом · м \)).

Поскольку проводимость является мерой того, насколько легко течет электричество, удельное электрическое сопротивление измеряет, насколько материал сопротивляется потоку электричества.

Свойства электротранспорта

Проще говоря, электричество — это движение электронов в материале. Когда электроны движутся через материал, он вступает в контакт с атомами в материале. Столкновения замедляют электроны. Каждое столкновение увеличивает удельное сопротивление материала. Чем легче электроны проходят через материал, тем меньше происходит столкновений и тем выше проводимость.

При повышении температуры проводимость металлов обычно уменьшается, а проводимость полупроводников увеличивается. Это, конечно, предполагает, что материал однороден, что не всегда так. Вы можете рассчитать удельное сопротивление, используя следующее уравнение

\ [\ dfrac {E} {J} = ρ \]

Как вы уже читали, ρ — это символ удельного сопротивления. \ (E \) представляет собой электрическое поле и измеряется в вольтах на метр (В / м). J — плотность тока, выраженная в амперах на квадратный метр (А / м2).Электрическое поле рассчитывается путем деления напряжения на длину l, к которой приложено напряжение.

\ [E = \ dfrac {V} {l} \]

Плотность тока рассчитывается по формуле ниже

\ [J = \ dfrac {I} {A} \]

I — это ток, деленный на площадь поперечного сечения A, по которой течет ток.

Сопротивление против сопротивления

Удельное сопротивление и сопротивление — это разные вещи. Удельное сопротивление не зависит от размера или формы.Однако сопротивление есть. Вы можете рассчитать сопротивление с помощью приведенного ниже уравнения.

\ [R = \ dfrac {V} {I} \]

R относится к сопротивлению и измеряется в Ом. \ (V \) — напряжение, измеряемое в вольтах. Я измеряю ток, и его единица измерения — амперы (А).

Список литературы

  1. Электропроводность и удельное сопротивление, Хини, Майкл, Электрические измерения, обработка сигналов и дисплеи. Июль 2003 г.

  2. Леви, Питер М., и Шуфэн Чжан. «Электропроводность магнитных многослойных структур». Physical Review Letters 65.13 (1990): 1643-646. Распечатать.

Проблемы

  1. Какова плотность тока материала с удельным сопротивлением 12 Ом · м и электрическим полем 64 В / м?
  2. Если напряжение 6 В проходит через вещество радиусом 2 м и длиной 3 м, что такое электрическое поле?
  3. Каково электрическое поле материала, когда ток равен 25 А, измеренное сопротивление составляет 78 Ом, плотность тока равна 24 А / м2, а длина протекает ток 100 м?
  4. Материал имеет напряжение 150 В и ширину 24 м.Материал также имеет ток 62 А и проходит расстояние 5 м. Какая проводимость?
  5. Металл изначально имеет электрон, сталкивающийся с каждым пятым атомом, и температура повышается с 6K до 100K. Полупроводник изначально имеет электрон, сталкивающийся с каждым пятым атомом, и температура повышается с 6K до 100K. Какой материал будет иметь большее удельное сопротивление? Почему?

Ответов на проблемы:

1. E / J = ρ —> J = E / ρ = 64 В / м / 12 Ом · м = 5.33А / м 2

2. E = V / l = 6V / 3m = 2V / m

3. E = об / л

В = ИК —> E = ИК / l = 25 А x 78 Ом / 100 м = 19,5 В / м

4. E / J = ρ

E = об / л

J = I / A —> ρ = (В / л) / (I / A) = (150 В / 5 м) / (62 A / (24 м x 5 м) = 58 Ом · м

ρ = 1 / σ —> 1 / ρ = σ = 1/58 Ом · м

5. Материал, имеющий наибольшее удельное сопротивление, — это металл, потому что с повышением температуры металлы с большей вероятностью будут увеличивать удельное сопротивление, а полупроводники обычно уменьшают удельное сопротивление при повышении температуры.

alexxlab

Добавить комментарий

Ваш адрес email не будет опубликован. Обязательные поля помечены *